You are on page 1of 269

Lời tựa

Trong tủ sách của KEM hiện nay (www.tapchikem.com), bạn có thể dễ


dàng tìm thấy hai bộ sách chính phục vụ cho việc tự học và bồi dưỡng
HSG Quốc gia là Hành trình Olympiad (2200 trang, thay thế cho bộ “Tự
học hóa”) và 55 Chuyên đề bồi dưỡng HSGQG (4400 trang). Cả hai bộ
sách này tuy đầy đủ về mặt kiến thức, nhưng tương đối khó để tiếp cận
– cả về nội dung lẫn giá thành – đặc biệt là với các độc giả mới chập
chững tìm hiểu về chương trình Hóa học phổ thông chuyên. Đó là lí do
series TOP100 ra đời: nhằm mang tới những ấn phẩm với GIÁ THÀNH RẺ
và nội dung DỄ TIẾP CẬN hơn cho các độc giả của KEM.
Quyển sách trên tay bạn là ấn phẩm đầu tiên về chủ đề Hóa học Hữu
cơ, với nội dung chính được chia thành 4 phần:
- Phần 1: Cấu trúc – Hoạt tính.
- Phần 2: Hydrocarbon
- Phần 3: Dẫn xuất halogen, oxygen và nitrogen.
- Phần 4: Hợp chất carbonyl và dẫn xuất.
Với các phần 2-4, bài tập được biên soạn theo hướng ứng dụng các kiến
thức từ phần 1, kết hợp với một số yếu tố mới lạ để giúp học sinh củng
cố vững chắc hơn nền tảng kiến thức. Đối tượng sử dụng phù hợp là
những học sinh đã tìm hiểu qua chương trình Hóa hữu cơ chuyên hoặc
sinh viên ngành Hóa.
Hi vọng rằng đây sẽ là một tài liệu hữu ích cho quý độc giả.
Trân trọng!
Ban biên tập Tạp chí KEM

1
Danh mục bài tập
Cấu trúc – Hoạt tính ....................................................................... 5
1. Lai hóa – Độ hơn cấp ............................................................................................ 5
2. Cấu tạo và danh pháp ........................................................................................... 7
3. Danh pháp IUPAC ................................................................................................. 9
4. Danh pháp IUPAC ............................................................................................... 11
5. Đồng phân hình học ............................................................................................ 13
6. Đồng phân quang học ......................................................................................... 14
7. Đồng phân quang học ......................................................................................... 15
8. Hóa lập thể ......................................................................................................... 19
9. Cấu trúc và cấu dạng .......................................................................................... 23
10. Cấu dạng .......................................................................................................... 26
11. Cấu dạng .......................................................................................................... 28
12. Cấu dạng .......................................................................................................... 29
13. Những vấn đề về cấu dạng ................................................................................ 30
14. Sự quay trục ..................................................................................................... 31
15. Phân tích FMO .................................................................................................. 32
16. Tiểu phân trung gian ......................................................................................... 33
17. Cộng hưởng ...................................................................................................... 35
18. Cộng hưởng ...................................................................................................... 38
19. Tính thơm ......................................................................................................... 44
20. Tính thơm ......................................................................................................... 46
21. Tính thơm ......................................................................................................... 48
22. Ảnh hưởng của tính thơm .................................................................................. 50
23. COT và [COT]2- .................................................................................................. 52
24. Ảnh hưởng của tính thơm .................................................................................. 53
25. Cấu tạo và hoạt tính .......................................................................................... 54
26. Cộng hưởng ...................................................................................................... 56
27. Nhiệt độ nóng chảy ........................................................................................... 59
28. Tính tan ............................................................................................................ 61
29. Tính tan ............................................................................................................ 63
30. Ứng dụng của tính tan ....................................................................................... 66
31. Tính acid .......................................................................................................... 70
32. Tính acid .......................................................................................................... 71
33. So sánh tính base ............................................................................................. 78
34. Tính thơm và tính acid ....................................................................................... 82
35. Tính acid và base .............................................................................................. 83
36. Sự hỗ biến ........................................................................................................ 86
37. Cấu tạo và hoạt tính .......................................................................................... 88
38. Hóa lập thể của phản ứng .................................................................................. 91
Hydrocarbon ................................................................................92
1. Phản ứng thế gốc................................................................................................ 92
2. Điều chế alkene .................................................................................................. 95
3. Hydrogen hóa alkene .......................................................................................... 96
4. Phản ứng cộng electrophile ................................................................................. 98
5. Phản ứng cộng electrophile ............................................................................... 101

2
6. Quy tắc Markovnikov ......................................................................................... 105
7. Cộng electrophile .............................................................................................. 109
8. Cộng electrophile .............................................................................................. 110
9. Đề xuất cơ chế phản ứng ................................................................................... 111
10. Chuyển vị carbocation ..................................................................................... 113
11. Chuyển vị carbocation ...................................................................................... 115
12. Tổng hợp linalool ............................................................................................ 117
13. Chuyển vị carbocation ..................................................................................... 119
14. Hydrobor hóa alkene ....................................................................................... 120
15. Oxi hóa alkene ................................................................................................ 122
16. Phản ứng ozone phân và oxi hóa phân cắt ........................................................ 125
17. Phản ứng của epoxi ........................................................................................ 129
18. Chuyển hóa hữu cơ ......................................................................................... 130
19. Xác định hydrocarbon ..................................................................................... 131
20. Xác định hydrocarbon ..................................................................................... 132
21. Xác định hydrocarbon ..................................................................................... 133
22. Sơ đồ chuyển hóa ........................................................................................... 134
23. Alkyne ............................................................................................................ 137
24. Alkyne ............................................................................................................ 141
25. Tổng hợp cyclopropane ................................................................................... 146
26. Hệ liên hợp ..................................................................................................... 147
27. Phản ứng Diels-Alder....................................................................................... 151
28. Phản ứng Diels-Alder....................................................................................... 154
29. Phản ứng vòng hóa Diels-Alder ........................................................................ 156
30. Hiện tượng đồng phân lập thể của các hợp chất vòng ....................................... 158
31. Ảnh hưởng của tính thơm ................................................................................ 161
32. Cơ chế phản ứng ............................................................................................. 164
Dẫn xuất halogen, oxygen và nitrogen ............................................ 165
1. Tổng quan về phản ứng thế nucleophile ............................................................. 165
2. Tính nucleophile ............................................................................................... 169
3. Ảnh hưởng của dung môi .................................................................................. 172
4. Ảnh hưởng của nhóm rời đi ............................................................................... 174
5. Hóa lập thể ....................................................................................................... 177
6. Sản phẩm phản ứng thế .................................................................................... 178
7. Sản phẩm phản ứng thế .................................................................................... 183
8. Phản ứng thế nucleophile .................................................................................. 185
9. So sánh tốc độ phản ứng................................................................................... 186
10. So sánh tốc độ phản ứng ................................................................................. 187
11. Cơ chế phản ứng ............................................................................................. 189
12. Cơ chế phản ứng ............................................................................................. 193
13. Hóa lập thể của phản ứng thế .......................................................................... 196
14. Xác định hợp chất hữu cơ ................................................................................ 199
15. Chuyển hóa hữu cơ ......................................................................................... 200
16. Chuyển hóa hữu cơ ......................................................................................... 202
17. Ảnh hưởng của nhóm kề .................................................................................. 204
18. Cơ chế phản ứng ............................................................................................. 208

3
19. Cơ chế phản ứng ............................................................................................. 210
19A. Cơ chế phản ứng........................................................................................... 212
20. Cấu dạng và phản ứng tách ............................................................................. 213
21. Tổng hợp thyroxine ......................................................................................... 216
22. Tổng hợp amine .............................................................................................. 221
23. Xác định amine ............................................................................................... 224
24. Xác định amine ............................................................................................... 225
Hợp chất carbonyl ....................................................................... 226
1. Phản ứng của các aldehyde và ketone................................................................ 226
2. Các hợp chất carbonyl ở dạng chất phản ứng và sản phẩm.................................. 230
3. Nhận biết hợp chất hữu cơ ................................................................................ 232
3. Tổng hợp hữu cơ............................................................................................... 234
3A. Điều chế nhóm chức ester ............................................................................... 235
4. Các enol và enolate trong hóa hữu cơ ................................................................ 237
4A. Phản ứng ngưng tụ ......................................................................................... 241
5. Chuỗi chuyển hóa ............................................................................................. 242
6. Xác định hợp chất tự nhiên ................................................................................ 245
7. Ngưng tụ benzoin ............................................................................................. 248
8. Tổng hợp hữu cơ............................................................................................... 251
9. Tổng hợp hữu cơ............................................................................................... 254
10. Cơ chế phản ứng ............................................................................................. 257
11. Cơ chế phản ứng ............................................................................................. 259
12. Cơ chế phản ứng ............................................................................................. 262
13. Cơ chế phản ứng ............................................................................................. 265
14. Cơ chế phản ứng ............................................................................................. 268

4
Chương 1

Cấu trúc – Hoạt tính


1. Lai hóa – Độ hơn cấp
1) Trong phân tử 4-vinyl-3-methylhepta-1,2-diene-5-yne có bao nhiêu
nguyên tử carbon lai hóa sp?

2) Xác định trạng thái lai hóa của các nguyên tử khác-hydrogen trong các
cấu trúc dưới đây.

3) Hãy xác định nhóm thế nào có độ hơn cấp cao hơn trong số các nhóm
sau đây:

5
Hướng dẫn
1) Có 3 nguyên tử carbon lai hóa sp.

2)

Chú ý rằng với chất B thì nguyên tử O nên được xem là lai hóa sp2 hơn là sp3
bởi theo đó thì cặp electron chưa liên kết trong orbital p sẽ tham gia vào sự
cộng hưởng với liên kết đôi hiệu quả nhất.
3)

6
2. Cấu tạo và danh pháp
i) Gọi tên gọi hợp chất CH3CH2C (CH3 )2 CH(CH3 )2 theo danh pháp IUPAC:
(A) 3,3,5-Trimethylhexane.
(B) 2,2,5-Trimethylhexane.
(C) 2,4,4-Trimethylhexane.
(D) 1,1,3,3-Tetramethylpentane.
(E) Không phải các phương án trên.
ii) Cấu dạng nào sau đây là của 2-methylpentane?

iii) Có bốn đồng phân cấu tạo ứng với công thức phân tử C4H9Br. Tên gọi
nào dưới đây là của một trong bốn đồng phân đó:
i) 1-Bromo-2-methylpropane.
ii) 3-Bromobutane.
iii) 2-Bromo-2-methylbutane.
iv) 2-Bromo-1-methylpropane.
iv) Gọi tên gọi hợp chất (CH3 )2 CHCH(CH3 ) CH2CH = CH2 theo danh pháp
IUPAC:
(A) 4,5-Dimethylhex-1-ene.
(B) 4,5,5-Trimethylpent-1-ene.
(C) 2,2-Dimethylhex-5-ene.
(D) 4-Methyl-4-isopropylbut-1-ene.

7
Hướng dẫn
i) 2,4,4-trimethylhexane (C).
ii) Cấu dạng C.
iii) Có 4 đồng phân cấu tạo ứng với công thức phân tử C4H9Br:

Trong số các tên gọi đã cho ở trên, chỉ có 1-bromo-2-methylpropane (A) là


đúng. Phương án 3-bromobutane (B) sai vì phải là 2-… mới đúng. Phương án
2-bromo-2-methylbutane (C) sai vì tên gọi này ứng với công thức phân tử
C5H11Br. Còn phương án 2-bromo-1-methylpropane (D) cũng không chính xác
vì chọn sai mạch dài nhất.
iv) Phương án (A).

8
3. Danh pháp IUPAC
Biểu diễn công thức cấu tạo của các hợp chất có tên gọi như sau:
a) 3-(iodomethyl)hexane
b) 3-fluoro-2,3-dimethylpentane
c) 5-(tert-butyl)-4-ethyl-2,3,4,7-tetramethyloctane
d) 2-(allyloxy)prop-1-ene
e) 1-cyclobutylcyclohept-1-ene
f) 3,3,5-trimethylcyclohexan-1-one
g) 5,6-dimethylhept-2-yne
h) 1,2-dichlorocyclobutane
i) 1-ethynyl-2-methylcyclopentane
j) 3,4-dimethylocta-1,5-diyne
k) 4-ethylheptan-3-ol
l) 1-ethoxy-3-ethylheptane
m) 3,4-dimethylocta-1,5-diyne
n) 4-ethylheptan-3-ol
o) 1-ethoxy-3-ethylheptane
p) 2-ethylhexane-1,4-diol
q) 3-isopropylhexan-2-ol
r) 2-ethoxy-1,1-dimethylcyclopentane
s) 2,2,2-tribromoacetaldehyde
t) 3-hydroxypentanal
u) 3,3,5,5-tetramethylheptan-4-one
v) (E)-3-chlorohexa-1,4-diene
w) (Z)-5-bromo-3-chlorohex-4-en-1-yne

9
Hướng dẫn

10
4. Danh pháp IUPAC
Gọi tên các hợp chất sau đây theo danh pháp IUPAC:

11
Hướng dẫn

12
5. Đồng phân hình học
i) Viết tất cả các cấu trúc của alkene có công thức C5H10 (bao gồm cả
các đồng phân lập thể). Gọi tên chúng theo danh pháp IUPAC.
ii) Cấu trúc nào sau đây là của (3E)-3,7-dimethylocta-1,3,6-triene?

iii) Xác định cấu hình các liên kết đôi của diene dưới đây:

(A) (2E,4E)
(B) (2Z,4Z)
(C) (2Z,4E)
(D) (2E,4Z)
Hướng dẫn
i)

ii) Cấu trúc D ứng với tên gọi (3E)-3,7-dimethylocta-1,3,6-triene.


iii) Cấu hình (2Z, 4Z) (B).

13
6. Đồng phân quang học
i) Liệu cis-4-tert-butylcyclohexan-1-ol hay trans-4-tert-butylcyclohexan-1-ol
có tính thủ tính?

ii) Hợp chất nào sau đây không có đồng phân quang học?

Hướng dẫn
i) Cả cis-4-tert-butylcyclohexan-1-ol lẫn trans-4-tert-butylcyclohexan-1-ol
đều không phải hợp chất thủ tính bởi phân tử của chúng đều có mặt phẳng
đối xứng.
ii) Chất C không có đồng phân quang học.

14
7. Đồng phân quang học
i) Vẽ công thức chiếu Fischer cho (S) và (R)-2-iodobutane.
ii) Xác định cấu hình tâm thủ tính của các phân tử sau:

iii) Vẽ công thức chiếu Fischer cho (2R, 3S)-2-bromo-3-chlorobutane và (2S,


3R)-2-bromo-3-chlorobutane, sắp xếp mạch carbon theo một đường
thẳng. Dựa vào các tính chất vật lí (nhiệt độ sôi, nhiệt độ nóng chảy), liệu
thể tách hỗn hợp đẳng mol của (2R,3S)-2-bromo-3-chlorobutane và
(2S,3R)-2-bromo-3-chlorobutane thành các hợp chất riêng biệt hay
không? Nếu có thì nên sử dụng kĩ thuật nào? Nếu không, hãy giải thích
nguyên nhân.
iv) Với mỗi cặp hợp chất sau, hãy chỉ ra mối quan hệ giữa chúng là đối
quang, xuyên lập thể phân (đồng phân dia), đồng phân cấu tạo hay cùng
là một chất?

15
v) Với mỗi cặp hợp chất sau, hãy chỉ ra mối quan hệ giữa chúng là: đồng
phân cấu tạo, xuyên lập thể phân, đối quang hay không phải các mối liên
hệ kể trên?

vi) Vẽ các đồng phân R và S của hợp chất dưới đây:

vii) Vẽ tất cả các đồng phân lập thể có thể có của hợp chất sau. Chỉ ra các
cặp đối quang và xuyên lập thể phân.

Hướng dẫn
i)

ii)

16
iii)

Các chất này là đối quang của nhau, nên chúng có cùng các tính chất vật lí
như nhiệt độ sôi hay nhiệt độ nóng chảy. Sự khác biệt chỉ liên quan đến sự
quay mặt phẳng ánh sáng phân cực. Do đó, không thể tách riêng chúng nếu
chỉ dựa vào các tính chất vật lí.
iv) A và B là cùng một chất, C và D là các xuyên lập thể phân, E và F là các
xuyên lập thể phân, G và H là các đối quang, I và J là các đối quang, K và L
là các đối quang, M và N là đồng phân cấu tạo.
v) a) Xuyên lập thể phân; b) Giống nhau; c) Đối quang; d) Xuyên lập thể phân.
vi)

Cách vẽ khác, cũng được chấp nhận:

17
vii)

18
8. Hóa lập thể
1) Methylphenidate (tên thương mại: Ritalin) được dùng trong điều trị
chứng rối loạn tăng động giảm chú ý (ADHD). Ritalin là hỗn hợp của các
đồng phân R,R và S,S mặc dù chỉ có đồng phân R,R là có hoạt tính điều
trị ADHD. (Đối quang tinh khiết R,R, gọi là dexmethylphenidate, hiện nay
được bán với tên thương mại Focalin.) Vẽ cấu trúc các đồng phân R,R và
S,S của methylphenidate.
2) Artemisinin và mefloquine được sử dụng rộng rãi trong điều trị sốt rét.
Hãy chỉ ra các tâm bất đối trong hai chất này và xác định cấu hình tuyệt
đối của chúng (*) theo R/S. Chỉ rõ trạng thái lai hóa của các nguyên tử
N trong mefloquine và cho biết nguyên tử nào bị proton hóa trong phản
ứng của mefloquine với HCl.

3) Saquinavir (tên thương mại Invirase) thuộc nhóm thuốc ức chế protease,
được sử dụng để điều trị HIV (virus gây suy giảm miễn dịch ở người).
Hãy chỉ ra các tâm bất đối trong Saquinavir và xác định cấu hình tuyệt
đối của chúng (*) theo R/S.

19
4) Vẽ đối quang của những hợp chất sau:

20
Hướng dẫn
1)

2)

Phản ứng của mefloquine với HCl:

21
3)

4)

22
9. Cấu trúc và cấu dạng
1) Vẽ cấu dạng bền của các hợp chất sau đây:

2) Người ta tìm thấy được một số gốc tự do A và B có cấu trúc như hình
dưới. Cho biết cấu trúc hình học của nguyên tử cacbon trung tâm trong
hai gốc này và giải thích tại sao nó có dạng hình học đó.

3) Trong cả ba trường hợp dưới đây, cấu dạng bên phải đều ưu thế hơn cấu
dạng bên trái:

3) Khi khảo sát hợp chất sau thì thấy rằng với nhóm OH nằm ở vị trí biên
(equatorial) thể hiện tính acid cao hơn so với lúc nhóm OH nằm ở vị trí
trục (axial). Hãy giải thích nguyên nhân.

4) Xét cân bằng cấu dạng của hai ketone dưới đây. Trong dung môi DMSO
thì cấu dạng A là cấu dạng ưu thế (100%), còn nếu sử dụng dung môi là

23
isooctane thì cấu dạng B lại là cấu dạng chủ yếu (22% A và 78% B). Giải
thích lý do.

Hướng dẫn
1)

2) Do không thể đạt được cấu trúc phẳng vì sự cứng nhắc của vòng nên cả
hai gốc tự do A và B đều có dạng hình tháp với nguyên tử cacbon trung tâm
lai hóa sp3
3)
- Ở cân bằng đầu tiên cấu dạng bên trái chịu sức căng 1,3-allyl bất lợi:

- Ở cân bằng thứ hai liên kết hydrogen tạo thành sẽ làm ổn định cấu
dạng bên phải:

- Ở cân bằng thứ ba cấu dạng bên trái chịu tương tác bất lợi giữa các
nhóm methyl và hydrogen:

24
4) Có thể giải thích theo hai hướng:
- Nguyên tử hydrogen của nhóm OH đã bị ràng buộc bởi liên kết
hydrogen nên khả năng phân li giảm.
- Anion sinh ra chịu đựng tương tác bất lợi với cặp electron tự do trên
nguyên tử O trong vòng làm giảm tính bền dẫn đến sự giảm tính acid
của chất đầu.

5) Ở cấu dạng A do tương tác lưỡng cực – lưỡng cực định hướng ngược nhau
nên A là cấu dạng chủ yếu trong dung môi có tính phân cực như DMSO. Còn
cấu dạng B mặc dù có tương tác không gian lớn (nhóm Me và i-Pr ở vị trí
axial) nhưng do có liên kết hydrogen trong dung môi không phân cực có thể
bù trừ cho tương tác lưỡng cực và tương tác không gian bất lợi, kết quả là
cấu dạng B ưu thế hơn trong dung môi không cực.

25
10. Cấu dạng
1) Hãy cho biết đồng phân nào bền hơn trong hai chất sau đây và giải thích
ngắn gọn

2) Hai hợp chất sau có hàm lượng enol tương ứng là 9.5% (Me2CHCHO chỉ
có 0.014%) và 95%. Hãy giải thích khả năng tồn tại ở dạng enol của các
chất này

3) Cho biết cấu dạng bền nhất của chất sau đây:

26
Hướng dẫn
1) Cấu dạng của đồng phân bền hơn:

Trong trường hợp này sẽ hạn chế tương tác 1,3-allylic đến mức tối thiểu
2) Giảm tương tác không gian khi tăng góc liên kết

3) Do có khả năng tạo thành các liên kết hydro nội phân tử giữa các nhóm
COOH nên cấu dạng bền nhất của triacid sẽ là:

27
11. Cấu dạng
1) Các haloethane (CH3CH2X, X = Cl, Br, I) có hàng rào năng lượng quay xấp
xỉ nhau (13.4 - 15.5 kJ/mol), mặc dù kích thước của các halogen tăng
theo chiều Cl < Br < I. Giải thích.
2) Khi hai vòng 6 cạnh có chung một liên kết C-C thì hệ bicylic này được gọi
là decalin. Có hai khả năng sắp xếp: trans-decalin có 2 nguyên tử
hydrogen ở phần ngưng tụ (C-C) nằm khác phía, và cis-decalin có 2
nguyên tử hydrogen ở phần ngưng tụ cùng phía. Giải thích tại sao đồng
phân trans- bền hơn?

Hướng dẫn
1) Mặc dù nguyên tử I lớn hơn nhiều so với Cl nhưng liên kết C-I cũng dài hơn
nhiều so với liên kết C-Cl. Kết quả là tương tác che khuất của các nguyên tử
H và I cũng không khác nhiều so với tương tác H, Cl.

2) Đồng phân cis kém bền hơn bởi tương tác trục (diaxial) 1,3 (xem hình.)

28
12. Cấu dạng
1) Chất nào trong số 2 chất sau đây phản ứng nhanh hơn với NaBH4? Giải
thích.

2) Hợp chất X dưới đây có khả năng tác dụng với acid để giải phóng
hydrogen. Hãy giải thích điều này dựa trên cấu trúc của nó.

Hướng dẫn
1) Cấu trúc decalin phản ứng nhanh hơn do sự chuyển dạng lai hóa từ Csp2
thành Csp3 dẫn đến sự giải tỏa tương tác che khuất (xem hình)

2) Cấu dạng của X được cho dưới đây. Lúc này


carbocation hình thành ở carbon trung tâm sẽ được
bền hóa mạnh do tương tác với ba cặp electron của
ba nguyên tử N kế cận khiến cho liên kết C - H trung
tâm rất dễ phân ly nên có thể tương tác được với acid
để sinh ra H2.

29
13. Những vấn đề về cấu dạng
1) Đối với 3-phenylpropanoic acid, năng lượng của cấu dạng mà Ph và
COOH che khuất nhau cao hơn so với cấu dạng mà Ph và COOH lệch
nhau (syn); cấu dạng mà Ph và H che khuất nhau; cấu dạng mà Ph đối
diện với COOH (anti) lần lượt là 15.0; 3.5; 18.5 kJ/mol. Hãy vẽ giản đồ
năng lượng cho sự chuyển đổi các cấu dạng nói trên.
2) Xếp theo thứ tự tăng dần sức căng vòng với ba hệ thống: cyclopropane,
oxirane và aziridine

Hướng dẫn
1) Giản đồ năng lượng chuyển đổi cấu dạng.

2) Khi tiến hành thay nhóm CH2 trong cyclopropane bằng các nhóm O hay
NH thì số tương tác che khuất H/H giảm do lúc này H được thay thế bằng
một cặp electron. Như vậy sức căng vòng sẽ tăng theo thứ tự oxirane <
aziridine < cyclopropane.

30
14. Sự quay trục
1) Xét quá trình quay quanh trục C-N sau đây trong các amide. Năng lượng
quan sát được của quá trình này là 20 kcal/mol. Giải thích

2) Đối với N,N-dimethylformamide thì các tính toán xác định năng lượng
trong phase khí cho kết quả là 19.4 kcal/mol, thấp hơn 1.5 kcal/mol nếu
tiến hành xác định trong dung dịch. Liệu cách giải thích ở ý (1) có thể áp
dụng cho trường hợp này hay không? Giải thích.
Hướng dẫn
Do ảnh hưởng của hiệu ứng liên hợp làm xuất hiện liên kết đôi C=N gia tăng
rào năng lượng, sự quay lúc này trở nên khó khăn hơn

Đối với DMF do cấu trúc lưỡng cực trong dung dịch bị solvate hóa nhiều hơn
nên khả năng quay sẽ bị ảnh hưởng nhiều hơn do còn phải có sự phá vỡ
solvate hóa.
Về sự thủy phân các amide này thì những hợp chất mà ở đó nguyên tử N bị
buộc vào một hệ thống căng hay hệ thống cứng nhắc thì khả năng liên hơp
N–C=O sẽ giảm nên các tác nhân (H+ hay OH-) dễ tấn công vào hơn trong
giai đoạn quyết định tốc độ phản ứng.

31
15. Phân tích FMO
Mercury(II) acetate là một tác nhân oxi hóa nhẹ nhàng có thể oxi hóa hai
muối ammonium bậc bốn I và II để tạo thành cation iminium tương ứng Ia
và IIa:

Một điều thú vị là tốc độ của phản ứng oxi hóa I nhanh gấp 13 lần so với tốc
k
độ của phản ứng oxi hóa II: I = 13. Giải thích kết quả này.
kII
Hướng dẫn

H và nhóm HgOAc nằm ở vị trí đối song, do đó sự tương tác giữa MO sigma
liên kết của C-H và MO sigma* phản liên kết của N-HgOAc diễn ra thuận lợi.

H và nhóm HgOAc không đối song với nhau, do đó sự tương tác giữa MO
sigma liên kết của C-H và MO sigma* phản liên kết của N-HgOAc diễn ra
không thuận lợi. Chính vì vậy mà có sự chênh lệch lớn về tốc độ giữa 2 phản
ứng trên.

32
16. Tiểu phân trung gian
i) Gốc tự do nào sau đây bền nhất?

ii) Carbocation nào sau đây bền nhất?

iii) Carbanion nào sau đây bền nhất?

iv) Carbocation (ion carbenium) bậc ba thì thường bền hơn carbocation bậc
một và bậc hai, bởi:
(A) có ba điện tích dương.
(B) có cấu hình dạng tháp.
(C) có cấu hình dạng tam giác phẳng.
(D) có ba nhóm đẩy electron.
v) Sắp xếp các carbocation sau theo thứ tự giảm dần tính bền:

33
vi) Vẽ cấu trúc gốc bền nhất được tạo thành từ sự tách một nguyên tử
hydrogen từ hợp chất dưới đây.

vii) Sắp xếp các hợp chất sau theo thứ tự tăng dần khả năng dễ phân li (đồng
li) thành các gốc tự do: 2,2,3,3-tetramethylbutane, 2,2-dimethyl-3,3,3-
triphenylpropane, hexaphenylethane, biphenyl.
Hướng dẫn
i) Gốc tự do bền nhất là B.
ii) Carbocation bền nhất là B.
iii) Carbanion bền nhất là A.
iv) Thường thì carbocation bậc ba sẽ bền hơn carbocation bậc một và bậc
hai bởi nó có ba nhóm alkyl đẩy electron (D).
v) Điện tích dương trên nguyên tử carbon có thể được bền hóa hiệu quả hơn
nhờ hiệu ứng cộng hưởng hơn là hiệu ứng siêu liên hợp. Do đó, cation benzyl
được bền hóa hơn tất cả các carbocation khác. Trong số các carbocation
còn lại thì càng nhiều nhóm alkyl sẽ càng giúp bền hòa điện tích dương, dẫn
đến tiểu phân càng bền.

vi) Gốc C dưới đây bền nhất:

vii) Độ bền của các gốc tự do giảm theo trình tự: Ph3C•  Me3C•  Ph•
Do đó, khả năng phân li thành các gốc tự do giảm theo trình tự sau:
Ph3C–CPh3  Ph3C–CMe3  Me3C–CMe3  Ph–Ph

34
17. Cộng hưởng
1) Vẽ cấu trúc cộng hưởng của mỗi chất sau đây:

2) Một trong các chất p-dinitrobenzene hoặc m-dinitrobenzene thường


được sử dụng làm chất bẫy gốc (radical trap) trong các phản ứng chuyển
electron. Hợp chất tạo thành anion gốc bền nhất sẽ là bẫy tốt hơn. Hãy
xét các anion gốc được tạo thành khi các chất đầu này nhận thêm một
electron và dự đoán hợp chất nào thường được sử dụng.
3) Theo bạn sự giải tỏa được thể hiện trong các cấu trúc cộng hưởng sau
có quan trọng không? Giải thích. Nếu điện tích dương được thay bằng
điện tích âm thì có gì khác không?

Hướng dẫn
1) a)

b)

35
c)

d)

e)

Dưới đây là một vai trong số nhiều cấu trúc có thể được vẽ ra:

f)

36
2) Bắt đầu bằng cách vẽ cấu trúc của các hợp chất liên hợp ban đầu rồi sau
đó thêm 1 electron vào để tạo thành anion gốc. Chỉ có một vài dạng cộng
hưởng khả dĩ được vẽ ra. Tuy nhiên có thể thấy rằng cả anion và gốc đều có
thể được giải tỏa đồng thời trên các nhóm nitro của p-dinitrobenzene, dẫn
đến nhiều dạng cộng hưởng khả dĩ hơn. Bởi trung gian tạo thành từ hợp chất
para được giải tỏa hơn nên sẽ dễ chuyển electron đến p-dinitrobenzene hơn
và nó sẽ là chất bẫy gốc tốt hơn.

3) Các dạng cộng hưởng với điện tích dương trên nitrogen hoặc oxygen
không đóng góp đáng kể vào sự bền hóa điện tích dương, bởi nitrogen và
oxygen không có “lớp vỏ bát tử” (8 electron) khi chúng mang điện tích
dương. Đây là điều cực kì quan trọng, khi oxygen và nitrogen có lớp vỏ bát
tử, chúng có thể đóng góp đáng kể vào dạng lai hóa cộng hưởng mà chúng
có điện tích dương. Ví dụ, carbocation 1-methoxyethyl được bền hóa đáng
kể bởi oxygen kế cận:

Nếu nitrogen và oxyen có điện tích âm, chúng sẽ có vỏ bát tử. Điện tích âm
trên các nguyên tử có độ âm điện lớn thế này sẽ được bền hóa đáng kể. Do
đó, sự giải tỏa như trên sẽ rất quan trọng trong trường hợp điện tích âm.

37
18. Cộng hưởng
1) Cấu trúc cộng hưởng nào mô tả trạng thái cơ bản của hợp chất sau tốt
hơn?

2) Vẽ cấu trúc cộng hưởng của các phân tử/ion sau:

3) Vẽ các cấu trúc cộng hưởng có thể có của mỗi hợp chất sau:

4) Các liên kết carbon-carbon trong naphthalene không bằng nhau. Sử dụng
cấu trúc cộng hưởng để giải thích tại sao liên kết (a) ngắn hơn liên kết
(b).

5) Khi xử lí indene với NaNH2 sẽ tạo thành base liên hợp của nó. Vẽ các cấu
trúc cộng hưởng có thể có của base liên hợp của indene và giải thích tại
sao pKa của indene lại thấp hơn của hầu hết các hydrocarbon.

38
6) Trong 5-methyl-1,3-cyclopentadiene (A) và 7-methyl-1,3,5-cyclohepta-
triene (B), nguyên tử H được đánh dấu nào có tính acid mạnh nhất?
Nguyên tử nào có tính acid yếu nhất? Giải thích tại sao.

7) a) Giải thích tại sao phản ứng proton hóa pyrrole diễn ra ở C2 để tạo
thành A thay vì ở trên 1 nguyên tử N để tạo thành B? b) Giải thích tại sao
A có tính acid hơn C, acid liên hợp của pyridine.

8) Giải thích tại sao hơn chất A bền hơn hợp chất B nhiều?

9) Giải thích tại sao triphenylene giống với benzene khi không xảy ra phản
ứng cộng với Br2 nhưng phenanthrene thì phản ứng được với Br2 tạo
thành sản phẩm cộng (như hình vẽ)? (Gợi ý: Vẽ cấu trúc cộng hưởng của
cả hai chất này và sử dụng chúng để xác định sự giải tỏa của các liên
kết π.)

39
10) Độ dài liên kết đơn C–C thường nằm trong khoảng từ 150 – 154 ppm, tuy
nhiên hợp chất có cấu trúc như dưới đây có một liên kết đơn C–C đặc
biệt dài. Giải thích tại sao nó lại dài đến vậy?

Hướng dẫn
1) Cấu trúc bên phải bởi các nguyên tử đều đạt bát tử.
2)

40
3)

Ở trường hợp cuối cùng thì cấu trúc thứ 2 có sức căng vô cùng lớn.
4) Naphthalene có thể có 3 cấu trúc cộng hưởng:

2 trong số các cấu trúc cộng hưởng có (a) là liên kết đôi, (b) là liên kết đơn.
Do đó, liên kết (b) có nhiều đặc trưng của liên kết đơn hơn nên nó dài hơn.
5) Base liên hợp của indene có 10 π electron nên nó có tính thơm và cực kì
bền. Do đó, tính acid của indene mạnh hơn hầu hết các hydrocarbon.

41
6)

Hb có tính acid mạnh nhất bởi base liên hợp của nó có tính thơm (6 π
electron). Hc có tính acid yếu nhất bởi base liên hợp của nó phản thơm (8 π
electron).
7) a) Sự proton hóa ở C2 tạo thành acid liên hợp A bởi điện tích dương có
thể được giải tỏa do cộng hưởng. Trong trường hợp B, điện tích dương không
được bền hóa cộng hưởng.

b) Việc tách 1 proton khỏi A (không có tính thơm) sẽ để lại electron cho N
và tạo thành pyrrole, có 6 π electron, được giải tỏa trong vòng 5 cạnh (hợp
chất có tính thơm). Do đó sự deproton hóa của A thuận lợi. Trong khi đó, cả
C và base pyridine liên hợp của nó đều có tính thơm. Do đó sự tách proton
ra không có nhiều tác động lớn đến tính thơm của hệ nên C có tính acid kém
hơn A. Thực tế thì pKa của A = 0.4 còn C = 5.2.
8) Cấu trúc cộng hưởng thứ hai của A cho thấy nó có vòng liên hợp hoàn
toàn với 6 π electron, vòng này có tính thơm và rất bền. Tương tự, với cấu
trúc B thì vòng liên hợp nhưng chỉ có 4 π electron (phản thơm), chính vì vậy
rất kém bền.

42
9) Các cấu trúc cộng hưởng của triphenylene:

Các cấu trúc cộng hưởng A-H đều có 3 liên kết đôi và 3 liên kết đơn trong 3
vòng 6 cạnh. Điều này có nghĩa là mỗi vòng đều có tính chất như một vòng
benzene cô lập, sẽ chỉ xảy ra phản ứng thế thay vì phản ứng cộng do mật độ
π electron được giải tỏa đều. Chỉ có duy nhất cấu trúc cộng hưởng I không
có dạng này. Mỗi liên kết C-C của triphenylene có 4 (hoặc 5) cấu trúc cộng
hưởng mà trong đó nó là liên kết đơn và 4 (hoặc 5) cấu trúc cộng hưởng mà
trong đó nó là liên kết đôi.
Các cấu trúc cộng hưởng của phenanthrene:

Tuy nhiên, với phenanthrene, 4 trong số 5 cấu trúc cộng hưởng có 1 liên kết
đôi ở các nguyên tử carbon được đánh dấu (chỉ có C là không.) Điều này có
nghĩa là 2 nguyên tử C này có nhiều đặc trưng liên kết đôi hơn các liên kết
C-C khác trong phenanthrene, do đó chúng dễ tham gia vào phản ứng cộng
thay vì phản ứng thế.
10) Tránh tương tác lập thể giữa hai nhân benzene, hỗ trợ tăng độ bền phân
tử.

43
19. Tính thơm
1) Cho biết mỗi tiểu phân sau đây có tính thơm, không thơm hay phản thơm:
a) b) c) d) f)

g) h) i) k)

2) Cho biết mỗi tiểu phân sau đây có tính thơm, không thơm hay phản thơm:
a) b) c) d) e)

f) g) h) i)

44
3) Cho biết mỗi tiểu phân sau đây có tính thơm, không thơm hay phản thơm:
a) b) c) d) e)

f) g) h) h) i)

Hướng dẫn
1) Thơm: c, d, e, h; Không thơm: f, g; Phản thơm: a, b, i.
2) Thơm: b, d, g, h; Không thơm: c, e, i; Phản thơm: a, f.
3) Thơm: a, c, d, e, g; Không thơm: b, h; Phản thơm: f, i.

45
20. Tính thơm
1) Hợp chất nào sau đây có tính thơm:

2) Hợp chất nào sau đây có tính phản thơm:

3) Hợp chất nào sau đây có tính thơm:

46
Hướng dẫn
1)

2)

3)

47
21. Tính thơm
1) Hợp chất nào sau đây có tính thơm:

2) Hợp chất nào sau đây có tính phản thơm:

48
Hướng dẫn
1)

2)

49
22. Ảnh hưởng của tính thơm
1) Các phân tử được liệt kê dưới đây có moment lưỡng cực cao bất thường.
Hãy giải thích tại sao.

2) Kết quả của các phản ứng sau là các sản phẩm kiểu-muối, bền (trong khí
quyển trở) được tạo thành. Hãy chỉ ra các sản phẩm này và giải thích
tính bền của chúng.

3) Phản ứng của azulene A với 1 đương lượng sulfuric acid sẽ tạo thành
hợp chất kết tinh C10H9  HSO4 − [B]. Hãy dự đoán cấu trúc của B.
+

50
Hướng dẫn
1) Moment lưỡng cực cao bất thường của các phân tử này (so với các alkene
và ketone thông thường) gây ra bởi các cấu trúc cộng hưởng có tính thơm,
thuận lợi về mặt năng lượng, với các điện tích tách biệt:

2)
Các tiểu phân tạo thành (A, B, C, D) đều là các cấu trúc có tính thơm, thuận
lợi về mặt năng lượng (bền).

3)

51
23. COT và [COT]2-
Cycloocta-1 3 5 7-tetraene (COT) được tổng hợp lần đầu tiên vào năm 1911.
Khi tương tác với potassium, COT nhận thêm 2 electron theo kiểu cộng 1,4
và trở thành dianion với 2 nguyên tử carbon mang điện tích âm, kí hiệu là
[COT]2-. Phương pháp nhiễu xạ tia X cho thấy COT có cấu trúc không phẳng,
khoảng cách giữa các nguyên tử cacbon cạnh nhau lần lượt là 1.33 và 1.46
Å.Trong khi đó, dianion [COT]2- có cấu trúc phẳng, khoảng cách giữa các
nguyên tử cacbon cạnh nhau đều bằng 1.41 Å.
1) Hãy vẽ cấu trúc dạng ghế và cấu trúc dạng thuyền của COT. So sánh độ
bền của hai cấu trúc này và giải thích.
2) 2.Vẽ công thức cấu tạo của [COT]2-. Vì sao nó có cấu trúc phẳng và có
các cạnh dài như nhau?
Hướng dẫn
1)

Dạng thuyền Dạng ghế


Dạng thuyền bền hơn dạng ghế, vì ở dạng thuyền, tất cả các liên kết đôi đều
có cấu tạo phẳng. Trong khi đó, ở dạng ghế, hai liên kết đôi giữa C3 với C4,
C7 với C8 không đồng phẳng, tạo ra sức căng, kém bền.
2) Với cấu trúc phẳng, [COT]2- bền vì trở thành hệ thơm (đó là một hệ liên hợp
vòng khép kín, số electron π thỏa mãn qui tắc Hückel 4n+2), các cạnh có độ
dài như nhau.

52
24. Ảnh hưởng của tính thơm
1) Hợp chất A tạo thành từ phản ứng dưới đây có moment lưỡng cực cao
bất thường (13.5 D). Xác định cấu trúc của A và lí giải cho giá trị moment
lưỡng cực của nó.

2) Cyclopentadiene-2,4-one, so với cyclohepta-2,4,6-trien-1-one, là hợp chất


rất kém bền và dễ bị dimer hóa ở nhiệt độ thường. Giải thích cho sự khác
biệt giữa hai chất này và viết cấu trúc dimer của cyclopentadien-2,4-one.
Hướng dẫn
1) Hợp chất A chứa các mảnh cationic và anionic có tính thơm.

2) Cyclohepta-2,4,6-trien-1-one có các cấu trúc cộng hưởng có tính thơm,


trong khi đó của cyclopentadiene-2,4-one lại là phản thơm.

53
25. Cấu tạo và hoạt tính
1) Mặc dù penicillin G có 2 nhóm amide nhưng có 1 nhóm hoạt động mạnh
hơn nhiều so với nhóm còn lại. Nhóm nào hoạt động mạnh hơn? Tại sao?

2) Xác định các tâm electrophilic và nucleophilic trong mỗi phân tử sau:

3) Chỉ bằng cách lập luận dựa vào mật độ electron, hãy xác định em liệu
các phản ứng sau có xảy ra hay không:

54
Hướng dẫn
1) Nhóm amide trong vòng 4 cạnh có góc liên kết 90o, dẫn đến sức căng
vòng lớn, do đó sẽ có hoạt tính hóa học cao hơn.
2)

- Trong d thì tất cả các liên kết C=C đều có tính nucleophile.
- Trong f thì tất cả các cặp electron chưa liên kết trên O và Cl đều có
tính nucleophile.
3)

55
26. Cộng hưởng
Giải thích các hiện tượng sau:
a) Các amide (R2NCOR) có tính nucleophile trên O mạnh hơn trên N.
b) Nhóm ester có tính electrophile của C kém hơn so với trong ketone.
c) Các acyl chloride (RCOCl) có tính acid mạnh hơn ester.
d) Hợp chất 1 có moment lưỡng cực lớn hơn đồng phân 2.
e) Hợp chất 3 có tính acid mạnh hơn 4.
f) Imidazole 5 có tính base mạnh hơn đáng kể so với pyridine 6.
g) Fulvene 7 có tính electrophile ở carbon gắn ngoài vòng (exocyclic).
h) Cyclohexadienone 8 dễ bị hỗ biến (tautomer hóa) hơn đa số các hợp chất
carbonyl. (Chú ý: Sự hỗ biến của các hợp chất carbonyl hầu hết đều diễn
ra rất nhanh, do đó câu hỏi này là về xu hướng nhiệt động học, không
phải động học.)
i) Cyclopentadienone 9 cực kì kém bền.
j) Chênh lệch pKa giữa PhSH và EtSH nhỏ hơn nhiều so với giữa PhOH và
EtOH.
k) Furan 10 chỉ tấn công electrophile ở C2, không ở C3.

Hướng dẫn
a) Cả N và O trong amide đều có các cặp electron chưa liên kết, có thể phản
ứng với electrophile. Khi O phản ứng với một tác nhân electrophile E+, sẽ tạo
thành sản phẩm có 2 cấu trúc cộng hưởng tốt. Còn khi N phản ứng chỉ tạo
thành sản phẩm có 1 cấu trúc cộng hưởng.
Phản ứng trên O: Phản ứng trên N:

56
b) Các ester có năng lượng thấp hơn ketone do sự bền hóa cộng hưởng từ
nguyên tử O. Khi cộng một tác nhân nucleophile vào ester hoặc ketone, một
tiểu phân trung gian tứ diện được tạo thành mà đối với nó sự cộng hưởng
gần như không quan trọng. Do đó, sản phẩm tứ diện từ ester có năng lượng
gần như giống với sản phẩm từ ketone. Kết quả là sẽ cần nhiều năng lượng
để cộng một nucleophile vào ester hơn là ketone.
c) Lập luận như trong ý b cũng có thể áp dụng vào trường hợp tính acid của
acyl chloride so với ester. Chú ý rằng Cl và O có độ âm điện gần như bằng
nhau, do đó chênh lệch về tính acid giữa acyl chloride và ester không phải
do hiệu ứng cảm ứng mà bởi hiệu ứng cộng hưởng.
d) Hợp chất 1 có một cấu trúc cộng hưởng mà trong đó các điện tích được
tách riêng. Thông thường, một cấu trúc có các phần mang điện tách riêng
thì có đóng góp nhỏ, nhưng trong trường hợp này, hai vòng được tạo thành
có tính thơm, do đó chúng quan trọng hơn bình thường.

e) Khác biệt giữa 3 và 4 là bởi chất đầu có vòng. Việc mất 1 H có tính acid
từ γ C của 3 tạo thành một tiểu phân có cấu trúc cộng hưởng thơm. 4 không
có tính chất tương tự.

f) Cả imidazole và pyridine đều là những hợp chất thơm. Cặp electron chưa
liên kết của N gắn với H trong imidazole là cần có để duy trì tính thơm, do đó
nguyên tử N còn lại, có cặp chưa liên kết trong orbital sp2, là tâm base. Sự
proton hóa nguyên tử N này tạo thành hợp chất có hai cấu trúc cộng hưởng
tốt. Còn sự proton hóa pyridine tạo thành một hợp chất thơm chỉ có duy nhất
một cấu trúc cộng hưởng tốt.

g) Các liên kết π C=C của các hydrocarbon đơn giản thường có tính
nucleophile, không có tính electrophile. Tuy nhiên, khi một nucleophile tấn
công vào một nguyên tử C ngoài vòng (exocyclic) của một hợp chất thơm

57
như fulvene thì các electron từ liên kết π C=C đi đến nguyên tử C trong vòng
(endocylic) và tạo thành hệ thơm.

không thơm thơm


h) Dạng hỗ biến của 2,4-cyclohexadienone, một hợp chất không thơm là
phenol, một hợp chất thơm.
i) Nhóm carbonyl C=O có hợp phần cộng hưởng C+-O- quan trọng. Trong
cyclopentadienone, hợp phần cộng hưởng này là cấu trúc phản thơm. (Chính
vì vậy, trong các đề xuất tổng hợp đừng lựa chọn cyclohexadienone hoặc
cyclopentadienone làm nguyên liệu đầu!)
j) PhOH có tính acid mạnh hơn đáng kể so với EtOH (pKa = 10 so với 17) do
sự bền hóa cộng hưởng của base liên hợp trong PhOH. S lớn hơn O, do đó
xen phủ S(p)-C(p) trong PhS- nhỏ hơn nhiều so với xen phủ O(p)-C(p) trong
PhO-. Sự xen phủ giảm xuống trong PhS- dẫn đến sự bền hóa cộng hưởng bị
gảm xuống, do đó, vòng phenyl gây ra khác biệt về tính acid của RSH ít hơn
so với ROH.
k) Sự tấn công của electrophile E+ vào C2 tạo thành một carbocation có 3
cấu trúc cộng hưởng tốt. Còn sự tấn công vào C3 chỉ tạo thành 2 cấu trúc
cộng hưởng tốt.

58
27. Nhiệt độ nóng chảy
Cho bảng số liệu sau:
Acid béo Stearic acid Palmitic acid Oleic acid Linoleic acid
tnc, oC 69.6 63.1 13.4 5.2
a) Biết công thức phân tử của oleic acid là C18H34O2 có chứa 1 liên kết đôi
ở dạng cis ở C9-C10 (C carboxyl là C1), công thức phân tử của linoleic
acid là C18H32O2 có chứa 2 liên kết đôi đều ở dạng cis ở C9-C10 và C12-
C13 (C carboxyl là C1). Hãy viết công thức cấu trúc của oleic acid và
linoleic acid.
b) Hãy nêu nguyên nhân dẫn tới sự giảm dần nhiệt độ nóng chảy của 4
acid đã cho.
c) Hãy giải tích vì sao chất béo thực vật thường có nhiệt độ đông đặc thấp
hơn chất béo động vật?

59
Hướng dẫn
Ở thể rắn các phân tử stearic acid có cấu trúc thẳng, gọn gàng, dễ sắp xếp chặt
khít nên lực hút giữa chúng mạnh hơn, trong khi đó các phân tử oleic acid có cấu
trúc uốn gập (ở chỗ cấu hình cis), cồng kềnh, khó sắp xếp chặt khít nên lực hút
giữa chúng yếu hơn. Hình dưới đây cho thấy cùng một thể tích sẽ chứa được
nhiều phân tử có cấu trúc thẳng hơn là phân tử có cấu trúc uốn gập:

Thành phần chính của chất béo thực vật là oleic acid vốn có tonc thấp.

60
28. Tính tan
Dự đoán tính tan trong nước của các phân tử hữu cơ sau:
a) DDT (thuốc trừ sâu, không phân b) Mestranol (thành phần trong
hủy được) thuốc tránh thai)

c) Aspartame (chất làm ngọt nhân d) Caffeine (chất kích thích trong
tạo tổng hợp) café, trà và nhiều loại nước giải
khát)

e) Succrose (đường cát) f) Carotatoxin (độc tố thần kinh


phân lập được từ củ cà rốt)

61
Hướng dẫn
a) DDT không có nitrogen, oxygen nên rất khó tan trong nước.
b) Caffeine có nhiều liên kết phân cực chứa nitrogen, oxygen và có khả
năng tạo nhiều liên kết hydrogen với nước nên có thể tan được.
c) Mestranol có 2 nhóm chức phân cực nhưng số carbon lớn (10 C) nên
không tan trong nước.
d) Sucrose tương tự như caffeine (tan).
e) Aspartame tương tự như caffeine (tan).
f) Carotatoxin tương tự như mestranol (không tan).

62
29. Tính tan
1) Dự đoán tính tan của các vitamin sau trong nước và trong dung môi hữu
cơ:
a) Vitamin E.

b) Vitamine B6 (pyridoxine)

2) Giải thích tại sao A kém tan trong nước hơn B, mặc dù cả hai hợp chất
này có các nhóm chức giống nhau.

3) Avobenzone và dioxybenzone là hai loại kem chống nắng thương mại.


Sử dụng những nguyên lí về tính tan, dự đoán loại kem chống nắng nào
dễ bị rửa trôi khi người dùng đi bơi. Giải thích lựa chọn của bạn.

63
Hướng dẫn
1) Về cơ bản, tính tan được xác định bởi độ phân cực. Các phân tử phân cực
tan được trong nước, trong khi đó các phân tử không phân cực tan được
trong dung môi hữu cơ.
a) Vitamin E:

- Chỉ có 2 nhóm phân cực.


- Có nhiều (29) liên kết C-C và C-H không phân cực.
 Tan được trong dung môi hữu cơ, khó tan trong nước.
b) Vitamin B6 (pyridoxine):

- có nhiều liên kết phân cực, ít liên kết không phân cực.
 Dễ tan trong nước.
(Ngoài ra, nó cũng có thể tan trong dung môi hữu cơ bởi bản chất hữu cơ
của mình, nhưng có khả năng là tan trong nước nhiều hơn.)
2) Trong A, các nhóm OH và CHO đủ gần nhau để tạo thành liên kết hydrogen
nội phân tử. Trong B, hai nhóm này xa nhau. Do hai nhóm chức phân cực
tham gia tạo liên kết hydrogen nội phân tử nên A khó tạo liên kết hydrogen
với nước hơn B. Điều này dẫn đến độ tan trong nước của A < B (có sẵn hai
nhóm chức để tạo liên kết hydrogen với dung môi H2O)

64
3) So sánh các nhóm chức trong 2 thành phần của kem chống nắng.
Dioxybenzone có khả năng bị rửa bởi nước nhất nó chứa 2 nhóm hydroxy và
tan trong nước nhiều hơn.
- Avobenzo: 2 nhóm ketone, 1 nhóm ether.
- Dioxybenzo: 2 nhóm hydroxyl, 1 nhóm ketone, 1 nhóm ether – tan
trong nước tốt hơn.

65
30. Ứng dụng của tính tan
1) Poly(ethylene glycol) (PEG) và poly(vinyl chloride) (PVC) là những ví dụ
tiêu biểu về các polymer - những phân tử hữu cơ lớn được tạo thành từ
nhiều đơn vị giống nhau nhỏ hơn, nối với nhau bởi liên kết cộng hóa trị.
Các polymer có những tính chất rất khác nhau phụ thuộc (một phần) vào
các nhóm chức của chúng. Hãy đánh giá tính tan trong nước của mỗi
polymer và giải thích tại sao PEG được dùng trong dầu gội còn PCV được
dùng để chế tạo các ống dẫn nước.

2) Tetrahydrocannabinol (THC) là thành phần hoạt tính trong cần sa, còn
ethanol là thành phần của đồ uống có cồn. Giải thích tại sao khi khám
sàng lọc thuốc có thể phát hiện ra sự hiện diện của THC nhưng không
phát hiện ra ethanol sau vài tuần kể từ khi các chất này được vào cơ thể?

3) Cocaine là một loại thuốc gây nghiện, bị lạm dụng nhiều. Cocaine thường
được điều chế ở dạng muối hydrochloride (cocaine hydrochloride)
nhưng có thể chuyển thành ma túy “crack” (phân tử trung hòa) bằng cách
xử lí với base. Chất nào trong số hai hợp chất này (xem hình) có nhiệt độ
sôi cao hơn? Chất nào tan trong nước tốt hơn? Dựa vào độ tan tương đối
hãy giải thích tại sao crack thường được sử dụng bằng cách hút (hít),
còn cocaine hydrochloride thì được bơm trực tiếp vào máu?

66
4) Không như xà phòng - là hợp chất ion - một số chất tẩy rửa dạng lỏng là
các phân tử trung tính. Giải thích cách mỗi phân tử sau đây hoạt động
để tẩy rửa vết bẩn như xà phòng.

a)

b)

Hướng dẫn
1) Do có các nguyên tử oxygen, PEG có khả năng tạo liên kết hydrogen với
nước, giúp cho PEG tan được trong nước và phù hợp cho các sản phẩm
như dầu gội. PVC không có liên kết hydrogen với nước, do đó PVC không
tan trong nước, mặc dù nó có nhiều liên kết phân cực. Do PCV không tan
trong nước nên có thể dùng để vận chuyển và giữ nước (ví dụ làm đường
ống).

2) Các phân tử tan trong nước thì dễ bị bài tiết ra khỏi cơ thể (qua nước
tiểu), trong khi đó các phân tử kém phân cực thì dễ hòa tan trong các
mô mỡ (dung môi hữu cơ) và được giữ lại trong thời gian dài hơn. Đây
chính là lí do tại sao khám sàng lọc có thể phát hiện được THC (không
tan trong nước) nhưng không phát hiện được ethanol sau vài tuần.

67
3) So sánh các lực hút liên phân tử giữa crack và cocaine hydrochloride. Các
lực hút liên phân tử mạnh hơn sẽ làm tăng cả nhiệt độ sôi lẫn độ tan trong
nước.

Các phân tử này giống nhau ngoại trừ liên kết ion trong cocaine
hydrochloride. Lực hút ion là lực cực kì mạnh, và do đó muối cocaine
hydrochloride có nhiệt độ sôi cao hơn và độ tan trong nước lớn hơn. Do muối
tan nhiều trong nước nên có thể tiêm trực tiếp vào máu. Crack được dùng
bằng cách hút (hít) do nó có thể tan trong các mô hữu cơ của mũi và phổi.
4) Một chất tẩy rửa cần phải có cả đầu phân cực lẫn đầu không phân cực.
Đầu phân cực sẽ tương tác với nước, còn đầu không phân cực sẽ bao xung
quanh mỡ và chất hữu cơ.
a)

68
Đầu không phân cực tương tác với Đầu phân cực tương tác với nước
chất hữu cơ qua các liên kết hydrogen
b)

Đầu không phân cực tương tác với Đầu phân cực tương tác với nước
chất hữu cơ qua các liên kết hydrogen

69
31. Tính acid
Sắp xếp (có giải thích) theo trình tự tăng dần tính acid của các hợp chất sau:
CH3CH2COOH ; CH3CH2CH(CH3 )OH ; CH3CHBrCOOH ; CH3CH2NH2
Hướng dẫn
Một cách để so sánh tính acid là so sánh khả năng phân ly cho proton H+,
khả năng này tùy thuộc vào liên kết H–X– và ảnh hưởng của các nhóm liên
kết với –X– của chất. Nếu các nhóm liên kết và bản chất của X làm cho liên
kết H–X– kém bền, dễ bị cắt đứt thì H trở nên linh động, khả năng phân li
cho proton càng dễ (tính acid càng mạnh). Trình tự tăng dần tính acid của
các hợp chất:

Giải thích:
- Tính acid của A yếu nhất vì: So sánh về độ âm điện của nguyên tố X
trong liên kết H-X- ; độ âm điện của oxygen lớn hơn nitrogen nên liên
kết -O-H phân cực mạnh hơn liên kết -N-H, hydrogen trong A kém linh
động hơn hydrogen trong các hợp chất khác trong dãy nên A có tính
acid yếu nhất.
- Tính acid của B, C, D: Chất C và D có hiệu ứng cảm ứng của nhóm
carbonyl làm O-H phân cực mạnh, đồng thời hiệu ứng liên hợp p - 
giúp giải tỏa điện tích âm của ion carboxylate. Chất B có gốc sec-
butyl đẩy electron làm giảm sự phân cực của liên kết O–H trong B
nên hydrogen kém linh động, C và D có tính acid mạnh hơn B.
- Tính acid của D mạnh hơn C vì: D có nguyên tử bromine hút electron
(-I) làm cho hydrogen của nhóm OH cũng linh động, nên có tính acid
mạnh hơn C.

70
32. Tính acid
Đánh giá lực acid của mỗi nhóm hợp chất sau và sắp xếp các chất trong mỗi
nhóm theo trật tự giảm dần lực acid.
a) HCO2H, CH3CO2H, CICH2CO2H, FCH2CO2H
b) Phenol, m- và p-chlorophenol, m- và p-cresol
c) Benzoic acid, m- và p-nitrobenzoic acid, m- và p-methoxybenzoic acid.
d) 1,4-Pentadiene and cyclopentadiene
e)

f)

g) CHF3 và CHCI3
h)

Hướng dẫn
a) Khi xét đến lực acid, yếu tố quan trọng nhất cần xét đến là độ bền của
base liên hợp so với acid. Nhìn chung, bất kì yếu tố nào làm tăng độ bền của
base liên hợp so với acid thì sẽ làm tăng tính acid. Các thuật ngữ hiệu ứng
cảm ứng và hiệu ứng cộng hưởng (mesomeric effect) được sử dụng xuyên
suốt trong các thảo luận về tính acid và tính base. Hiệu ứng đầu tiên tác
động qua khung σ của phân tử, còn hiệu ứng sau thì qua hệ π.
Lực acid tương đối được đo bởi các giá trị pKa. Lực acid của một phân tử là
phép đo liệu nó dễ mất proton tới mức nào; phân tử có lực acid càng mạnh
thì mất proton càng nhanh. Với một acid dạng HA, chúng ta có thể viết cân
bằng sau:

71
HA H+ + A −
Trong một hợp chất có lực acid rất mạnh, đa số các proton có tính acid sẽ
được giải phóng và cân bằng chuyển dịch sang bên phải. Nồng độ của H+ và
A- cao, còn nồng độ của HA thấp. Do đó, hằng số cân bằng Ka (
H+   A − 
Ka = ) lớn. Ngược lại, nếu một hợp có lực acid yếu, chỉ có một vài
HA 
proton được tách ra và cân bằng chuyển dịch sang bên trái. Do đó, hăng số
cân bằng nhỏ.
Do khoảng hằng số cân bằng rất lớn nên Ka thường không được sử dụng để
đo trực tiếp tính acid, thay vào đó các giá trị pKa thường được dẫn ra. pKa
của một hợp chất là logarithm âm của giá trị Ka (pKa = -log10Ka). pKa nhỏ thì
ứng với lực acid mạnh, pKa lớn thì lực acid yếu. Cần lưu ý là do sử dụng thang
logarith, nên một thay đổi nhỏ cũng thể hiện biến đổi hằng số cân bằng lớn
(bởi pKa thay đổi 1 đơn vị là hằng số cân bằng đã biến đổi 10 lần.)
a) HCO2H, CH3CO2H, ClCH2CO2H, FCH2CO2H
Xu hướng biến đổi tính acid trong dãy các hợp chất X-CO2H thể hiện hiệu ứng
cảm ứng của nhóm thế X. Anion tạo thành bởi sự mất 1 proton được bền hóa
bởi các nhóm hút electron như fluorine và chlorine nhưng sẽ bị làm kém bền
bởi các nhóm nhường electron như methyl. Do đó, FCH2CO2H có lực acid
mạnh nhất trong dãy (pKa = 2.58), sau đó là ClCH2CO2H (pKa = 2.86), HCO2H
(pKa = 3.75) và CH3CO2H (pKa = 4.76).
b) Phenol, m- và p-chlorophenol, m- và p-cresol
Phenol (pKa = 10.0) có lực acid mạnh hơn các alcohol không thơm do anion
tạo thành bởi sự mất 1 proton được bền hóa qua sự giải tỏa điện tích âm
khắp vòng benzene. Hiện tượng này được minh họa bởi các cấu trúc cộng
hưởng sau đây:

Lực acid của các phenol thế có thể được giải thích dựa vào ảnh hưởng của
nhóm thế trên mỗi cấu trúc cộng hưởng. Anion được tạo thành bởi sự mất 1
proton từ chlorophenol được bền hóa bởi hiệu ứng cảm ứng âm (hút

72
electron) từ nguyên tử chlorine, nên các chlorophenol có lực acid mạnh hơn
phenol. Tuy nhiên, tình huống này phức tạp hơn thế: bởi nguyên tử chlorine
với cặp lone pair (cặp electron chưa liên kết) có hiệu ứng liên hợp dương
(đẩy electron) với vòng benzene và làm giảm độ bền anion1. Tác động này
trở nên rõ ràng với p-chlorophenol (pKa = 9.38) hơn m-chlorophenol (pKa =
9.02).

Trong cấu trúc cộng hưởng thứ 3, lực đẩy điện tử giữa lone pair trên chlorine
và nguyên tử carbon mang điện tích âm rõ ràng không thuận lợi, và đóng góp
của cấu trúc này rõ ràng làm năng lượng bền hóa cộng hưởng giảm đi.
Trong m-chlorophenol, hiệu ứng liên hợp dương có tác động làm bền hóa
nhỏ hơn do không có cấu trúc cộng hưởng tương đương nào với điện tích
âm trên meta carbon.

Trong các crezol (methylphenol), hiệu ứng cảm ứng dương của nhóm methyl
làm giảm độ bèn của anion, nên các cresol có lực acid tháp hơn phenol.
Tương tự như hiệu ứng liên hợp trong chlorophenol, hiệu ứng cảm ứng dương
này trở nên rõ ràng với p-crezol (pKa = 10.27) hơn m-cresol (pKa = 10.9).
c) Benzoic acid, m- và p-nitrobenzoic acid, m- và p-methoxybenzoic acid.
Trong benzoic acid (pKa = 4.20), anion được tạo thành bởi sự mất 1 proton
được bền hóa bởi sự giải tỏa điện tích âm trên 2 nguyên tử oxygen tương
đương - được thể hiện qua 2 cấu trúc cộng hưởng dưới đây. Trái ngược với
phenol, điện tích âm không thể được bền hóa bằng cách giải tỏa vào vòng
benzene. Do lực acid của benzoic acid lớn hơn phenol nhiều, nên có thể kết

1
Các tác động ngược nhau của hiệu ứng cảm ứng âm và sự liên hợp nhường electron trong
các benzene thế halo- cũng được quan sát thấy trong các phản ứng thế electrophile nhân
thơm của chúng.

73
luận rằng sự giải tỏa trong hợp phần carboxylic acid hiệu quả hơn sự giải tỏa
cộng hưởng của phenoxide anion vào vòng thơm.

Nhóm nitro là nhóm hút electron, và đo đó cũng làm bền hóa anion nên các
nitrobenzoic acid có lực acid mạnh hơn benzoic acid. p-Nitrobenzoic acid
(pKa = 3.44) có lực acid mạnh hơn một chút so với m-nitrobenzoic (pKa =
3.45). Điều này không có gì bất ngờ bởi sự tồn tại của cấu trúc cộng hưởng
thứ ba dưới đây, với điện tích dương liền kề nhóm carboxylate giàu electron
- bền hơn bất kì cấu trúc cộng hưởng tương tự nào của đồng phân meta. Chú
ý rằng với mỗi cấu trúc cộng hưởng dưới đây, đều tồn tại một cấu trúc cộng
hưởng tương đương của nhóm carboxylate.

Nhóm methoxy, cũng như nhóm thế chloro, có hiệu ứng cảm ứng âm (hút
electron) nhưng lại có hiệu ứng liên hợp dương (nhường electron). Trái
ngược với nhóm thế chloro, hiệu ứng liên hợp dương của nguyên tử oxygen
mạnh hơn hiệu ứng cảm ứng âm nhiều. (Mặc dù độ âm điện của 2 nguyên tử
tương đương nhau, nhưng bán kính nguyên tử nhỏ hơn của oxygen giúp sự
xen phủ orbital với nguyên tử carbon tốt hơn, do đó hiệu ứng liên hợp mạnh
hơn.) Các methoxybenzoic acid có lực acid yếu hơn benzoic acid. Lực acid
của p-methoxybenzoic acid (pKa = 4.47) yếu hơn một chút so với m-
methoxybenzoic acid (pKa = 4.09) do cấu trúc cộng hưởng thứ ba (dưới đây),

74
có nguyên tử carbon mang điện tích âm cạnh nhóm carboxylate anion - kém
bền hơn bất kì cấu trúc tương đương nào của đồng phân meta.

d) 1,4-Pentadiene và cyclopentadiene.
1,4-Pentadiene (pKa = 35) có lực acid mạnh hơn hydrocarbon no tương ứng,
vì anion tạo thành bởi sự mất 1 proton được bền hóa cộng hưởng:

(Việc đo pKa của 1,4-pentadiene không dễ thực hiện, do anion này tồn tại
trong cân bằng với cả 1,4-pentadiene lẫn 1,3-pentadiene [bền về mặt nhiệt
động học hơn]. Do đó, giá trị pKa trên là tính toán được chứ không phải đo
trực tiếp.)
Phân tử phẳng cyclopentadiene (pKa = 15.0) thậm chí còn có lực acid mạnh
hơn bởi sự mất 1 proton sẽ tạo thành hệ vòng 6 electron (thơm), được bền
hóa cộng hưởng tương tự như benzene:

e) Trong mỗi hợp chất trên, proton có lực acid mạnh nhất đều thuộc về
nguyên tử carbon đánh dấu hoa thị (*) và anion tương ứng được bền hóa bởi
sự tạo thành enolate ion, có thể được biểu diễn bởi các cấu trúc cộng hưởng
sau đây:

75
Cả 3 nhóm thế R đều khiến cho anion bền hơn so với dạng enolate không
liên hợp, bởi điện tích âm có thể được giải tỏa tiếp. 2 nhóm keto làm cho
anion có độ bền lớn nhất (pKa = 8.9) bởi sự giải tỏa trên 5 nguyên tử, gồm 2
nguyên tử oxygen âm điện.

Hợp chất cyano có lực acid nhỏ hơn (pKa = 10.0) do nitrogen có độ âm điện
thấp hơn oxygen:

Nhóm thế ester làm bền hóa kém nhất (pKa = 11.0) do sự giải tỏa điện tích
âm vào nhóm carbonyl (ester) lại được bị giảm chút bởi hiệu ứng liên hợp
dương của lone pair từ oxygen:

f) Trong phân tử đầu tiên (pKa > 40), sự tạo thành enolate ion rất không thuận
lợi: khi loại bỏ 1 proton, orbital có điện tích âm nằm vuông góc với các
carbonyl π orbital. Dạng hình học này ngăn cản sự xen phủ orbital để tạo
thành một hệ π mở rộng:

76
Cấu trúc cộng hưởng II (dưới đây) - vi phạm quy tắc Bredt - có đóng góp rất
nhỏ bởi dạng hình học của phân tử không thuận lợi cho sự xen phủ carbon-
carbon π.

Với trường hợp thứ hai, enolate ion có thể được tạo thành và hợp chất này
có lực acid mạnh hơn (pKa = 20):

g) CHF3 và CHCI3
Do độ âm điện của fluorine lớn hơn nhiều so với chlorne nên F3C- được dự
đoán là bền hơn nhiều so với dẫn xuất chlorine tương ứng, và CHF3 có thể có
lực acid mạnh hơn CHCl3 rát nhiều. Tuy nhiên, thực tế thì CHCl3 có lực acid
lớn hơn, do Cl3C- được bền hóa bởi sự giải tỏa điện tích âm trong các d orbital
của chlorine. Trái lại, nguyên tử fluorine không có d orbital nên F3C- kém bền
hơn nhiều. Do đó, lực acid của CHF3 (pKa = 32) thấp hơn CHCl3 (pKa = 24)
khá nhiều.
h) Dù có độ âm điện thấp hơn nhưng hợp chất của sulfur (pKa = 35) lại có
lực acid mạnh hơn chất tương ứng chứa oxygen (pKa > 40). Điều này được
giải thích bởi khả năng mở rộng vỏ bát tử của sulfur (sử dụng các orbital d),
tạo thành các cấu trúc cộng hưởng sau:

77
33. So sánh tính base
So sánh tính base của các hợp chất nitrogen trong mỗi cặp chất sau (xét
trong dung dịch nước). Giải thích câu trả lời dựa trên các hiệu ứng lập thể
hoặc các yếu tố không gian.
a) b)

c) d)

e) f)

g) h)

i)

78
Hướng dẫn
a)

N-methylaniline là base yếu hơn benzylamine do hiệu ứng cộng hưởng +M


của nhóm amino liên kết trực tiếp với vòng benzene. Trong benzylamine,
hiệu ứng này không được thể hiện, do nhóm amino tách biệt với vòng thơm
(cách bởi nhóm -CH-). Đồng thời, benzylamine là base yếu hơn methylamine
(pKa 10.62, pKb 3.38) do hiệu ứng -I của nhóm C6H5.
b)

Cyclohexylamine có tính base mạnh hơn sáu bậc (106) so với aniline do hiệu
ứng cộng hưởng +M của nhóm amino trên vòng benzene.
c)

Khác biệt giữa tính base giữa pyrrolidine và pyrrole là khoảng 15 bậc (1015).
Thực tế thì pyrrolidine cũng là một amine bậc hai phổ biến, và trong pyrrole
thì cặp electron chưa liên kết của nitrogen cũng là một phần thuộc hệ thơm
(6 electron) của dị vòng, do đó không tham gia phản ứng proton hóa – phản
ứng này được diễn ra ở vị trí thứ hai của vòng, ở nguyên tử carbon.

79
d)

p-toluidine có tính base mạnh hơn bốn bậc so với p-nitroaniline, do nhóm
nitrogen làm gia tăng mức độ liên hợp của cặp electron trên nguyên tử
nitrogen của nhóm nitro với vòng benzene, dẫn đến tính base giảm mạnh.
e)

Quinuclidine là amine bậc ba được cố định bởi hệ bicyclic, có tính base


mạnh. Trong pyridine thơm, cặp electron chưa liên kết của nguyên tử
nitrogen thuộc orbital lai hóa sp2, nằm vuông góc với hệ thơm (6 electron
pi). Do đó, pyridine có tính base thấp hơn 5 bậc so với quinuclidine, có cặp
electron thuộc orbital lai hóa sp3.
f)

Aniline có tính base mạnh hơn bởi cặp electron chưa liên kết của nó tham
gia vào hệ liên hợp (hiệu ứng +M) với một vòng thơm, trong khi đó
diphenylamine thì là với hai vòng thơm.

80
g)

Piperidine có tính base mạnh hơn pyridine khoảng 6 bậc, do cặp electron
chưa liên kết của nitrogen thuộc orbital lai hóa sp3, trong khi đó của pyridine
là sp2.
h)

Tính base của p-hydroxyaniline và p-toluidine tương đối gần nhau (chênh
lệch dưới 10 lần). n-hydroxyaniline (p-aminophenol) có tính base mạnh hơn
một chút do hiệu ứng +M của nhóm OH làm giảm hiệu ứng +M của nhóm
amino, dẫn đến tính base của nguyên tử nitrogen trong chất này mạnh hơn.
i)

1,8-bis (dimethylamino naphthalene (còn gọi là "bọt xốp (hấp thụ) proton")
là base khá mạnh, do ở dạng proton hóa thì hydrogen tạo thành liên kết
hydrogen mạnh với nguyên tử nitrogen của nhóm amino liền kề.

81
34. Tính thơm và tính acid
Giải thích sự khác nhau về tính acid C-H của các proton methylene trong các
hợp chất sau:

pKa 22.9 18.5 15 36

Hướng dẫn
Tính acid C-H của các proton methylen trong các hợp chất này liên quan đến
độ bền của carbanion, tức là các base liên hợp, được tạo thành khi tương
tác với base. Trong hợp chất B, tính acid của các proton methylene mạnh
nhất, do sự deproton hóa (tách proton) dẫn đến sự tạo thành anion
cyclopentadienyl thơm; còn trong hợp chất D, có tính acid yếu nhất, thì khi
deproton hóa tạo thành anion tropylium phản thơm. Các hợp chất A và B, với
tính acid nằm ở khoảng giữa, thì về mặt hình thức có mảnh cyclopentadienyl
trong cấu trúc, nhưng sự bền hóa carbanion tạo thành sẽ phá vỡ hệ thơm
bền của các vòng benzene.

82
35. Tính acid và base
1)
a) Fumaric acid và maleic acid có các hằng số phân ly nấc 1 (k1), nấc 2
(k2). Hãy so sánh các cặp hằng số phân ly tương ứng của hai acid
này và giải thích.
b) Tại sao squaric acid lại có pK2 nhỏ của cả fumaric acid và maleic
acid?

2) Giải thích tại sao:


a) Hợp chất dưới đây có tính acid rất mạnh (thậm chí còn mạnh hơn cả
nitric acid):

b) Tính base của chất B mạnh hơn A:

c) 1,1,3,3-tetramethylguanidine (pKa ≈ 12,00) là base mạnh hơn N,N-


dimethylacetamide (pKa ≈ 0,50) rất nhiều.

83
3) Puberulic acid (tên gọi IUPAC: 4,5,6-trihydroxy-3-oxocyclohepta-1,4,6-
triene-1-carboxylic acid) là chất kháng sinh có trong một loài nấm, vừa
có tính acid vừa có tính base. Hãy giải thích nguyên nhân xuất hiện tính
acid và tính base của puberulic acid.

Hướng dẫn
1) a) k1(M) > k1(F) là do M có khả năng tạo được liên kết hydrogen nội phân
tử, liên kết O-H của M trong qúa trình phân ly thứ nhất phân cực hơn so với
F và base liên hợp M’ cũng bền hơn F’.

k2(M) < k2(F) do liên kết hydrogen nội phân tử làm cho M’ bền, khó nhường
proton hơn so với F’. Ngoài ra base liên hợp M’’ lại kém bền hơn (do năng
lượng tương tác giữa các nhóm –COO- lớn hơn) base liên hợp F’.
b) pK2 của squaric acid nhỏ hơn pK2 của fumaric acid và maleic acid là do
dianion của squaric acid tạo được hệ thống liên hợp bền vững:

2)

84
a)

Anion sinh ra được bền vững bằng hiệu ứng siêu liên hợp âm của nhóm CF3,
nhưng nhân tố quan trọng hơn đóng góp vào tính bền của anion chính là sự
tạo thành hệ thơm bền vững (6e).
b)

Trong cấu trúc A có sự liên hợp giữa cặp electron trên N và nhóm C=O làm
giảm tính base. Điều này không tồn tại trong B do với cấu trúc vòng cứng
nhắc không thể tồn tại sự xen phủ hiệu lực giữa obitan chứa cặp e trên N với
nhóm CO, vì thế tính base của B mạnh hơn.
c) Do cation sinh ra từ guanidine được ổn định mạnh bằng cộng hưởng.
3) Chất này acid vì có nhóm -COOH và các nhóm- OH dạng enol nên có khả
năng phân li tạo thành H+. Có tính base vì có khả năng nhận H+ tạo
cacbocation bền là vòng có tính thơm (theo qui tắc Hückel).

85
36. Sự hỗ biến
1) Tại sao pKa của các proton Ha trong 1-acetylcyclohexene cao hơn pKa
của các proton Hb?

2) Tại sao 5,5-dimethyl-1,3-cyclohexanedione chủ yếu tồn tại ở dạng enol


nhưng 2,2-dimethyl-1,3-cyclohexanedione thì không?

86
Hướng dẫn
1) Sự tách loại Ha tạo thành 2 cấu trúc cộng hưởng mà điện tích âm không
bao giờ ở trên O. Sự tách loại Hb tạo thành 3 cấu trúc cộng hưởng mà một
trong số đó có điện tích âm ở trên O, làm cho base liên hợp bền hơn. Chính
vì vậy Hb có tính acid mạnh hơn (pKa thấp hơn).

2) 5,5-dimethyl-1,3-cyclohexanedione chủ yếu tồn tại ở dạng enol bởi liên kết
C=C của enol liên hợp với nhóm C=O khác. Sự liên hợp làm bền hóa enol này.
Enol của 2,2-dimethyl-1,3-cyclohexanedione không liên hợp với nhóm
carbonyl còn lại.

87
37. Cấu tạo và hoạt tính
1) So sánh độ dài liên kết C=O trong ba hợp chất sau đây và giải thích ngắn
gọn:

2) Giải thích tốc độ dung môi phân của các hợp chất sau đây trong acetic
acid:
X Tốc độ tương đối
H 1
OMe 130
Me 5
Cl 0.2
3) Giải thích hướng của phản ứng sau đây:

4) Khi hoàn tan đồng phân cis và trans của hợp chất dưới đây vào acid
sunfuric 60% thì chỉ mỗi đồng phân cis tạo dung dịch màu vàng đậm,
còn đồng phân trans cho dung dịch trong suốt. Hãy giải thích điều này.

88
Hướng dẫn
1) Độ dài liên kết các chất tăng dần theo thứ tự:

Để so sánh độ dài liên kết ta phải xét khả năng cộng hưởng của cặp e trên
dị tố N vào C=O. Ở đây A không thể tham gia cộng hưởng (vi phạm quy tắc
Bredt) nên liên kết C=O hoàn toàn là liên kết đôi nên A phải có liên kết C=O
ngắn nhất. C dài hơn A nhưng ngắn hơn B do khả năng tham gia cộng hưởng
của N tốt hơn O, thành ra liên kết C=O trong B có nhiều tình chất liên kết đơn
hơn nên dài hơn.
2) Phản ứng dung môi phân có sự tạo thành carbocation không cổ điển (hình
vẽ), nó được ổn định hóa bởi các nhóm dồn electron dẫn đến bảng kết quả
như trên

3)

Phản ứng tạo thành carbocation không cổ điện được ổn định hóa bằng liên
kết đôi qua tương tác không gian. Điều này dẫn đến việc mặt sau của liên
kết C - OTs đã bị chắn nên phản ứng chỉ có thể xảy ra theo hướng bảo toàn
cấu hình.

89
4) Ở đồng phân cis tạo thành carbocation được ổn định bằng obitan pi của
liên kết đôi, cộng thêm việc nhóm phenyl trợ màu dẫn đến tạo thành dung
dịch có màu. Đồng phân trans không có sự hỗ trợ từ liên kết đôi bên vòng
cyclohexene nên không có màu:

90
38. Hóa lập thể của phản ứng
i) Phản ứng chlor hóa (S)-2-methyl-1-chlorobutane, được kích phát bởi tia
tử ngoại, dẫn đến sự tạo thành một hỗn hợp mà từ đó có thể phân lập
được 2-methyl-1,2-dichlorobutane và 2-methyl-1,3-dichlorobutane.Dựa
vào quan điểm của bạn, hãy đánh giá hóa lập thể của các sản phẩm tạo
thành.
ii) Dẫn ra các công thức chiếu Fischer cho các đồng phân 2,3-
dichlorobutane tạo thành khi chlor hóa racemic 2-chlorobutane dưới ánh
sáng tử ngoại. Chỉ ra cấu hình R, S của mỗi nguyên tử carbon bất đối.
Hướng dẫn
i) Các gốc tự do không bảo toàn cấu hình tứ diện. Do đó, phản ứng
monochlor hóa (S)-2-methyl 1-chlorobutane quang hoạt ở vị trí số 2 sẽ tạo
thành 2-methyl-1,2-dichlorobutane dạng racemic. Khi chlor hóa vị trí số 3 thì
cấu hình của carbon C2 được bảo toàn, dẫn đến sự tạo thành hỗn hợp xuyên
lập thể phân của (2R, 3R)- và (2R, 3S)-2-methyl-1,3-dichlorobutane:

ii)

91
Chương 2

Hydrocarbon
1. Phản ứng thế gốc
i) Xác định công thức sản phẩm chính của phản ứng bromo hóa
ethylcyclohexane và đề xuất cơ chế tạo thành nó.

ii) Trong mỗi cặp chất sau, hãy chỉ ra hợp chất có nguyên tử hydrogen
(được kí hiệu bởi các chữ cái) có hoạt tính mạnh hơn trong phản ứng
halogen hóa gốc.
a) b)

iii) Phản ứng bromo hóa các alkene diễn ra chậm hơn nhiều so với chlor
hóa. Xác định sản phẩm chính của phản ứng giữa 2-methylbutane với
bromine khi chiếu sáng hoặc đun nóng.

iv) Xác định sản phẩm chính của phản ứng monobromo hóa 2,2-
dimethylbutane.

92
v) Sắp xếp các hợp chất trong dãy sau theo thứ tự tăng dần hoạt tính trong
phản ứng bromo hóa cơ chế gốc.

vi) Hợp chất nào sau đây không được tạo thành trong phản ứng bromo hóa
gốc tự do 1-ethyl-4-methylbenzene.

Hướng dẫn
i) Phản ứng diễn ra qua gốc bậc ba bền hơn.

ii) а) Hа > Hb; b) Hc > Hd.

93
iii) Sản phẩm chính trong phản ứng của 2-methylbutane với bromine khi
chiếu sáng hoặc đun nóng là bromide B. Do hoạt tính của bromine không
quá mạnh nên phản ứng bromo hóa diễn ra có độ chọn lọc cao, alkyl bromide
được tạo thành chủ yếu là bởi sự thế diễn ra ở liên kết C-H hoạt tính nhất (dễ
phân li ra gốc bền nhất).

iv) Sản phẩm chính là hợp chất C, do độ chọn lọc của phản ứng bromo hóa
cao và sự tác kích của bromine diễn ra theo hướng tạo thành gốc bền nhất.

v) D  A  B  C. Thứ tự hoạt tính có mối tương quan với độ bền trung gian
gốc. Gốc tạo thành từ hợp chất D không được bền hóa bởi sự liên hợp với
vòng thơm.

vi) Trong phản ứng bromo hóa gốc tự do của 1-ethyl-4-methylbenzene không
thể tạo thành hợp chất D, là sản phẩm bromo hóa vào vòng benzene.

94
2. Điều chế alkene
i) Phản ứng nào sau đây không phù hợp để điều chế but-1-ene.

ii) Đề xuất cách điều chế trans-2-chlorocyclohexanol đi từ


chlorocyclohexane.
Hướng dẫn
i) Phản ứng B không phù hợp bởi theo quy tắc Zaitsev thì sản phẩm chính
tạo thành sẽ là but-2-ene.
ii)

95
3. Hydrogen hóa alkene
i) Hợp chất nào sau đây có nhiệt hydrogen hóa cao nhất?

ii) Trong dãy các bicycloalkene sau, hãy chọn ra hợp chất giải phóng nhiệt
lượng lớn nhất trong phản ứng hydrogen hóa.

iii) Hãy chọn ra điều kiện tối ưu nhất để thực hiện chuyển hóa của 1-
(chloromethyl)cyclohex-1-ene thành (chloromethyl)cyclohexane.

(A) H2 /Pt − C /EtOH


(B) 1. NaBH4 /EtOH, 2. H3O+
(C) 1. LiAlH4 /Et2O, 2. H3O+ .
(D) 1. (CH3 )2 CHCH2  AlH, 2. H3O+ .
2

iv) Phản ứng hydrogen hóa xúc tác một hydrocarbon thủ tính C6H12 tạo
thành sản phẩm là hydrocarbon phi thủ tính C6H14. Hydrocarbon C6H12
ban đầu là
(A) Cis-hex-2-ene
(B) 3-Methylpent-2-ene
(C) 4-Methylpent-2-ene
(D) 3-Methylpent-1-ene

96
Hướng dẫn
i) Hydrogen hóa là phản ứng tỏa nhiệt. Alkene càng có nhiều nhóm alkyl liên
kết với các nguyên tử carbon lai hóa sp2 thì càng bền, dẫn đến nhiệt hydrogen
hóa càng thấp. Độ bền tương đối của các alkene:

Giá trị nhiệt hydrogen hóa thực tế (theo kcal/mol) của các hợp chất đã cho
là: 26.6 ( C )  26.9 ( D )  27.6 ( A )  28.6 (E )  30.3 (B ).

ii) Hợp chất A giải phóng nhiệt lượng lớn nhất khi hydrogen hóa hoàn toàn
bởi alkene này kém bền nhất (giải thích tương tự ý i ở trên).
iii) H2 / Pt − C / EtOH ( A )

iv) 3-Methylpent-1-ene (D)

97
4. Phản ứng cộng electrophile
i) Hợp chất nào sau đây không phải là electrophile:
AlCl3 , C2H5OC2H5 , BF3 , (CH3 )3 C+ , HOCl.

ii) Alkene nào sau đây có thể phản ứng nhanh nhất với các tác nhân
electrophile?
(A) Н2С = СН2
(B) ( СH3 )2 C = CH2
(C) Cl2C = CCl2
(D) CF3C = CH2
iii) Dự đoán sản phẩm chính của phản ứng cộng bromine dưới đây, chỉ rõ
hóa lập thể:

iv) Đề xuất cấu trúc sản phẩm chính của phản ứng bromo hóa (1E)-but-1-en-
1-ylcyclohexane.

v) Xác định cấu trúc sản phẩm chính của phản ứng sau:

vi) Xác định sản phẩm chính của phản ứng sau:

98
Hướng dẫn
i) C2H5OC2H5

ii) ( СH3 )2 C = CH2 (B) có thể phản ứng với các tác nhân electrophile nhanh
nhất bởi nó có nhiều các nhóm methyl nhường electron quanh liên kết đôi.
iii) Phản ứng cộng hợp electrophile vào propylene dẫn đến sự tạo thành hỗn
hợp racemic của hai đối quang.

iv)

v) Phản ứng cộng nước bromine vào methylenecyclopentane là một ví dụ về


phản ứng cộng electrophile vào liên kết đôi với sự tham gia của một tác nhân
nucleophile “bên ngoài”. Phản ứng này được gọi là phản ứng cộng “liên hợp”.
Trong phản ứng của một alkene không đối xứng, halogen luôn được gắn vào
nguyên tử carbon có số hydrogen lớn nhất, còn tác nhân nucleophile (nước)
vào nguyên tử carbon với số hydrogen nhỏ hơn.

99
vi) Vi hạt có tính electrophile là I+ do nguyên tử chlorine âm điện hơn. Tác
nhân electrophile sẽ tác kích vào liên kết đôi để tạo thành carbocation bậc
hai bền hơn.

100
5. Phản ứng cộng electrophile
i) Tại sao phản ứng sau lại tạo thành 1,2-dibromocyclopentane ở dạng hỗn
hợp tiêu triền (racemic)?

ii) Sản phẩm nào không được tạo thành trong phản ứng dưới đây:

iii) Dự đoán sản phẩm chính của các phản ứng sau:
a) b)

c) d)

e) f)

g)

101
iv) Xác định các tác nhân và điều kiện phù hợp với mỗi chuyển hóa sau của
1-methylcyclohex-1-ene?

(A) H2O, 60%H2SO4 , 0 − 10 oC


(B) H2 , PtO2
(C) 1.Hg (OCOCH3 )2 ,THF,CH3OH; 2.NaBH4 ,CH3OH
(D) 1.BH3 ,THF; 2.H2O2 ,NaOH,H2O
(E) HBr,h, (PhCOO)2
(F) HBr,CH3COOH
(G) Br2 ,CCl4 ,0 − ( −5 ) C
o

102
Hướng dẫn
i) Sơ đồ phản ứng bromine hóa cyclopentene tạo thành 1,2-dibromocyl
clopentane:

ii) Các hợp chất C và D không được tạo thành trong phản ứng của 1-
methylcyclopentene với nước bromine. Electrophile trong phản ứng này là
Br+, còn nucleophile là Br- và H2O.
iii) a) Đây là phản ứng cộng electrophile, diễn ra qua trung gian carbocation
bền hơn.

b) Trong trường hợp này nucleophile là nước. Phản ứng không sử dụng
sulfuric acid đặc nên khó xảy ra sự tách nước.

c) Phản ứng hydrobor hóa-oxi hóa (cộng hợp phản Markovnikov):

d) Do nguyên tử chlorine âm điện hơn nên nguyên tử bromine thể hiện các
tính chất electrophile. Sự tác kích diễn ra bởi anion chloride ưu tiên diễn ra
ở nguyên tử carbon có xu hướng tạo thành carbocation bậc ba bền hơn.

103
e) Acid sẽ proton hóa liên kết đôi có nhiều nhóm thế hơn, dẫn đến sự tạo
thành carbocation bền hơn.

f) Nucleophile là methanol.

g)

iv)

104
6. Quy tắc Markovnikov
Năm 2018 đã diễn ra lễ kỉ niệm 180 năm ngày sinh nhà Hóa học vĩ đại
Vladimir Vasilievich Markovnikov. Markovnikov đã dành sự nghiệp khoa học
của mình để nghiên cứu về các mô hình cộng hợp các hợp chất khác nhau
vào liên kết đôi C=C, bao gồm hợp chất A (ω(H) = 0.788%). Ví dụ, ông đã
nhận thấy rằng tương tác của hợp chất A với hydrocarbon B (ω(C) = 85.71%;
khi oxi hóa với potassium permanganate trong điều kiện khắc nghiệt sẽ tạo
thành sản phẩm hữu cơ duy nhất C, đi kèm với sự giải phóng CO2) sẽ tạo
thành hợp chất X. Dựa trên phản ứng này, Markovnikov đã đưa ra một nhận
xét mà về sau trở nên nổi tiếng với tên gọi “quy tắc Markovnikov”, và sản
phẩm tạo thành trong phản ứng này được gọi là Markovnikovsky.
1) Phát biểu quy tắc Markovnikov.
2) Xác định các hợp chất A-C và X, biết rằng C không chứa các nguyên tử
carbon bậc ba và để trung hòa C cần 17.5 mL dung dịch NaOH 0.500 M.
Thay vì hợp chất A, việc sử dụng các hợp chất khác có liên quan, ví dụ như
D (ω (H) = 2.76%) khiến Markovnikov gặp một vài khó khăn. Về sau các nhà
nghiên cứu nhận ra rằng tương tác của B với D tạo thành sản phẩm
Markovnikovsky E diễn ra trong khoảng 6-7 tháng. Để tăng tốc độ phản ứng
này, vào năm 1990, các nhà khoa học Hoa Kì đã đề xuất sử dụng hợp chất F
(ω(O) = 13.45%) cùng với oxide ướt G (ω (O) = 47.07%) làm tác nhân phản
ứng.
3) Xác định các hợp chất D-G.
Xử lí 3-chloropropionic acid với SOCl2, sau đó cho sản phẩm H phản ứng với
alkene đơn giản nhất I khi có mặt AlCl3 khan. Hợp chất đối xứng J tạo thành
được chưng cất trên sodium carbonate khan, tạo thành sản phẩm không
vòng đối xứng K. Dưới tác động của khí A, thay vì tạo ra sản phẩm
Markovnikow như dự đoán thì lại thu được hợp chất L chỉ chứa C, H, O (trong
trường hợp này, các nguyên tử carbon trong L tạo thành một vòng năm
cạnh). Nếu phản ứng tiếp tục thì khí A dư sẽ từ từ cộng hợp vào L và tạo
thành sản phẩm hữu cơ duy nhất M.

4) Xác định cấu trúc các hợp chất H-M. Giải thích tại sao khi A cộng hợp
vào L thì chỉ tạo thành duy nhất một sản phẩm hữu cơ, mặc dù trong

105
trường hợp này, quy tắc Markovnikov không thể đưa ra một câu trả lời rõ
ràng cho việc sản phẩm cộng nào sẽ chiếm ưu thế.
Hướng dẫn
1) Theo Bách khoa toàn thư Hóa học (Chemical Encyclopedia), quy tắc
Markovnikov được phát biểu như sau: Khi các protic acid (acid phân li H+)
hoặc nước cộng hợp vào một alkene hoặc alkyne bất đối xứng, thì nguyên tử
hydrogen sẽ gắn vào nguyên tử carbon có nhiều hydrogen hơn.
2) Nếu giả sử rằng hợp chất A chứa một nguyên tử hydrogen thì khối lượng
mol của nó là M(A) = 1.01 / 0.00788 = 128.2 g / mol, rất dễ để đoán được A
chính là HI.
Tiếp theo, ta xác định công thức tối giản của hydrocarbon B. Với B thì số mol
carbon (C) : (H) = 85.71 / 12.01: 14.29 / 1.01 = 7.137: 14.15 = 1: 1.98 ≈ 1:
2. Do đó, công thức của B là CnH2n. Do đề bài liên quan đến quy tắc
Markovnikov, rõ ràng B là một alkene, không thể là cycloalkane. Khi alkene
bị oxi hóa bởi potassium permanganate trong điều kiện khắc nghiệt thì xảy
ra sự phá vỡ liên kết C=C, tạo thành ketone, carboxylic acid hoặc CO2 - tùy
thuộc vào cấu trúc của alkene. Trong trường hợp này, CO2 được giải phóng
trong phản ứng oxi hóa alkene với liên kết đôi ở đầu mạch.
Do C thể hiện tính acid nên nó là carboxylic acid và là acid đơn chức, do
dicarboxylic acid chỉ có thể được tạo thành trong phản ứng oxi hóa
cycloalkene hoặc diene, chứ không phải alkene. Từ kết quả chuẩn độ, có thể
xác định số mol acid ban đầu: (C) = (NaOH) = C(NaOH) · V (NaOH) = 0.50
· 0.0175 = 0.00875 mol. Khi đó M(C) = 0.77 / 0.00875 = 88 g/mol, tương ứng
với công thức phân tử C4H8O2. Do C không chứa các nguyên tử carbon bậc
ba nên nó là butyric acid (butanoicacid, và hydrocarbon B là pent-1-ene. Khi
đó, X là 2-iodopentane.

3) Tương tự như trường hợp xác định chất A, M(D) = 1.01 / 0.0276 = 36.6 g
/ mol, tương ứng với hydrogen chloride HCl. Thực tế, phản ứng của B với HCl
cần thời gian tương đối lâu, bởi liên kết H-Cl mạnh hơn liên kế H-I (do đó
trong thực tế thì HI thể hiện tính acid mạnh hơn HCl). Phản ứng tạo thành 2-
chloropentane E:

106
Bây giờ, chúng ta xác định các hợp chất F và G. Để tính được thành phần
oxide G, công thức của nó có thể được biểu diễn là Y2Ox. Hàm lượng của
oxygen được cho trong đề bài có thể được biểu diễn theo ω(O) = 0.4707 =
16.00 · x / (16.00 · x + 2y), trong đó y là khối lượng mol của nguyên tố Y. Có
thể dễ dàng tính được y = 9x và chỉ có trường hợp Y là Al là phù hợp. Vậy
công thức oxide G là Al2O3.
Giả sử rằng F chứa 1 nguyên tử oxygen, khi đó M (F) = 16.00 / 0.1345 = 119.0
g / mol. Rõ ràng, chất F phải chứa chlorine, và do khối lượng mol của F là số
nguyên nên F phải chứa số nguyên tử chlorine chẵn. Nếu số nguyên tử
chlorine trong F là 2 thì phần còn lại của các nguyên tố (ngoại trừ O và Cl) là
119 - (71 + 16) = 32 g / mol - tương đương với 1 nguyên tử lưu huỳnh. Do đó
F là SOCl2.
Phản ứng cộng HCl bởi tác động của SOCl2 và Al2O3 được giải thích bởi thực
tế rằng việc sử dụng Al2O3 ướt dẫn đến sự thủy phân trực tiếp SOCl2 trong
hỗn hợp phản ứng, dẫn đến sự tạo thành HCl. Sự gia tăng tốc độ phản ứng
trong điều kiện này được giả định rằng liên kết H-Cl bị suy yếu bởi sự tạo
thành liên kết hydrogen với bề mặt aluminum oxide.
4) Khi xử lí 3-chloropropionic acid với SOCl2, thì tạo thành acid chloride H,
chất này cũng có thể được xác định bởi hàm lượng chlorine.
Alkene đơn giản nhất là ethylene (I).
Giả sử hợp chất J chứa một nguyên tử chlorine, thì M(J) = 35.45 / 0.4574 =
77.50 g / mol. Các nguyên tố có thể có còn lại (C, H, O) chiếm 77.50 - 35.45
= 42.05 g / mol - giá trị này có thể tương ứng với C3H6 hoặc C2H2O. Tuy nhiên,
không có phương án nào trong số này phù hợp (ít nhất là bởi nếu có một
nguyên tử chlorine thì J phải chứa số nguyên tử hydrogen lẻ). Nếu J chứa
hai nguyên tử chlorine thì M(J) = 35.45 · 2 / 0.4574 = 155.0 g / mol. Các
nguyên tố có thể có còn lại (C, H, O) chiếm 155.0 - 35.45 · 2 = 84.11 g / mol
- giá trị này có thể tương ứng với C5H8O (biến thể với 5 nguyên tử carbon là
hợp lí, do các chất phản ứng lần lượt chứa 3 và 2 nguyên tử carbon). Do đó,
công thức phân tử của J là C5H8Cl2O, bằng đúng tổng công thức phân tử của
các tác nhân. Do đó, trong phản ứng, acid chloride H đã cộng hợp qua liên
kết C=C của ethylene, tạo thành 1.5-dichloropentan-3-one (J) đối xứng.

107
Khi xử lí với base (Na2CO3), đun nóng dẫn đến sự phân cắt hai phân tử HCl,
tạo thành penta-1.4-diene-3-one K (cũng có thể dự đoán được sự hiện diện
của các liên kết C=C trong K trên cơ sở rằng phản ứng của K với HI có thể
tạo thành sản phẩm Markovnikov).
Bây giờ, hãy thiết lập công thức phân tử của L. Nếu L chứa một nguyên tử
oxygen, thì M(L) = 16.00 / 0.1949 = 82.09 g / mol, tương ứng với công thức
C5H6O. Hợp chất K có cùng công thức phân tử, vậy là dưới tác động của HI,
nó vòng hóa để tạo thành cyclopent-2-enon L theo cơ chế dưới đây (phản
ứng này được gọi là vòng hóa Nazarov). Cũng cần chú ý rằng cấu trúc L cũng
có thể được thiết lập mà không cần xét đến cơ chế tạo thành nó, do với hợp
chất có vòng carbon năm cạnh có công thức phân tử C5H6O, chúng ta chỉ có
thể đề xuất duy nhất một cấu trúc mà phản ứng cộng HI trong giai đoạn tiếp
theo có thể được chọn lọc.
L phản ứng với HI dư, chỉ tạo thành sản phẩm 3-iodocyclopentanone M. Sự
chọn lọc vị trí của phản ứng trong trường hợp này được giải thích bởi các
tính chất hút electron mạnh của nhóm carbonyl.

108
7. Cộng electrophile
1) Xác định sản phẩm tạo thành khi cộng hợp ClCH=CHOC2H5 với HCl (a)
và Br2 khi có mặt LiBr* đánh dấu đồng vị (b).
2) Xác định sản phẩm tạo thành khi cộng hợp HCl (a) và Br2 khi có mặt
LiBr* đánh dấu đồng vị (b) với:
CF3CH = CHCH2 ( CH3 )2 A
CH3CH = CHCOOH B 
Hướng dẫn
1)

2)

109
8. Cộng electrophile
1) Xác định sản phẩm được tạo thành trong phản ứng cộng ICl (a) và
Hg(OAc)2 trong methanol (sau đó demercury hóa) (b) của các hợp chất
sau:
CH2 = CHCOOEt  A  1-phenylcyclohexene B

2) Xác định sản phẩm được tạo thành trong phản ứng cộng HCl (a) và Cl2
trong nước (b) của các hợp chất sau:
N  CCH = CHOCH3  A  EtCH = CHPh B

Hướng dẫn
1)

2)

110
9. Đề xuất cơ chế phản ứng
1) Xử lí pent-4-ene-1-ol với bromine trong carbon tetrachloride thu được
một hợp chất có công thức phân tử C5H9OBr. Biểu diễn cấu tạo sản phẩm
và đề xuất cơ chế khả dĩ cho chuyển hóa này.
2) Dự đoán cấu tạo sản phẩm A và đề xuất một cơ chế khả dĩ cho quá trình
tạo thành sản phẩm này:

A có thể có bao nhiêu đồng phân lập thể?


3) Khi xử lí heptadiene-1,6 với nước bromine thì nhận được một hợp chất
có cấu tạo như sau:

Hãy đề xuất cơ chế tạo thành hợp chất này.

111
Hướng dẫn
1)

2)

A có hai nguyên tử carbon bất đối nên có thể có bốn đồng phân lập thể.
3)

112
10. Chuyển vị carbocation
i) Xác định công thức cấu tạo sản phẩm cộng nước của tert-butylethylene
và isopropylethylene trong môi trường acid. Trình bày cơ chế tạo thành
các sản phẩm đó.

ii) Xác định sản phẩm chính và đề xuất cơ chế của phản ứng:

Hướng dẫn
i)

113
ii) Chuyển hóa của 2,2-dimethylcyclohexanol trong môi trường acid đi kèm
với chuyển vị như sau:

114
11. Chuyển vị carbocation
Hợp chất 2,2,4-trimetylpentan (A) được sản xuất với quy mô lớn bằng
phương pháp tổng hợp xúc tác từ C4H8 (X) với C4H10 (Y). A cũng có thể được
điều chế từ X theo hai bước: thứ nhất, khi có xúc tác axit vô cơ, X tạo thành
Z và Q; thứ hai hydro hóa Q và Z
a) Viết các phương trình phản ứng để minh họa và gọi tên các hợp chất X,
Y, Z, Q theo danh pháp IUPAC
b) Ozon phân Z và Q sẽ tạo thành 4 hợp chất, trong đó có axeton và
fomandehit, viết cơ chế phản ứng
Hướng dẫn
a)

Bước thứ nhất gồm tương tác giữa hai phân tử trong môi trường axit

115
b)

116
12. Tổng hợp linalool
Linalool - một hợp chất tạo hương phổ biến, được dùng trong nhiều loại mỹ
phẩm: xà phòng, nước hoa, dầu gội đầu, … Các nhà nghiên cứu ước lượng
rằng khoảng 60-80% các loại sản phẩm như trên có chứa hợp chất hữu cơ
này, do đó việc tổng hợp thương mại linalool đóng vai trò quan trọng để đáp
ứng nhu cầu của thị trường, thay cho nguồn linalool tự nhiên khan hiếm.
1) Vẽ cấu trúc 3,7-dimethylocta-1,6-dien-3-ol (linalool).
2) Dùng dấu hoa thị để đánh dấu các nguyên tử carbon thủ tính.
Dưới đây là một sơ đồ tổng hợp linalool:

3) Xác định các sản phẩm khác có thể được tạo thành trong phản ứng cộng
của HBr vào isoprene. Vẽ cấu trúc của chúng.
4) Đề xuất các tác nhân B và C phù hợp với sơ đồ trên.
5) Đề xuất cơ chế phản ứng giữa A và alkene.
Một người học hóa có cái nhìn tốt về cấu trúc linalool có thể nhận thấy rằng
hợp chất này có thể chuyển thành vòng 6 cạnh trong các điều kiện phù hợp.
Mọi người đều biết rằng các tình nguyện viên tổ chức Olympiad khoa học
Lithuania (thường được gọi là các oranžiniai - hay người da cam) thực sự rất
thích cam. Cam có mùi dễ chịu bởi limonene - một sản phẩm vòng hóa của
linalool.
6) Đề xuất điều kiện phản ứng (trung tính/acid/base), tác nhân và cơ chế
cho chuyển hóa này.

117
Hướng dẫn
1-2)

3)

4)

5)

6) Môi trường có tính acid sẽ phù hợp. H2SO4 có thể đóng vai trò là một acid
lẫn base yếu (anion) để có thể thực hiện deproton hóa trong bước cuối của
cơ chế. Cơ chế đề xuất:

118
13. Chuyển vị carbocation
Đề xuất cơ chế phản ứng:

Hướng dẫn

119
14. Hydrobor hóa alkene
i) Alcohol nào được tạo thành trong phản ứng hydrobor hóa 1-
methylcyclohexene?

ii) Đề xuất cơ chế cho phản ứng ở ý i.


iii) Xác định sản phẩm chính của phản ứng sau và chỉ rõ hóa lập thể:

Hướng dẫn
1) Sản phẩm tạo thành là B (2-methylcyclohexanol) ở dạng hỗn hợp tiêu
triền.

ii) Phản ứng hydrobor hóa 1-methylcyclohexene, sau đó xử lí hỗn hợp phản
ứng với H2O2 /NaOH/H2O là phương pháp để tạp thành các alcohol từ
alkene. Giai đoạn đầu tiên của phản ứng hydrobor hóa là quá trình cộng hợp
phản Markovnikov, nghĩa là boron được gắn vào nguyên tử carbon ít nhóm
thế hơn. Sự oxi hóa liên kết C-B tạo thành alcohol.

120
iii)

121
15. Oxi hóa alkene
i) Xác định sản phẩm chính của phản ứng giữa 1,2-dimethylcyclohexene
với meta-chloroperbenzoic acid.

ii) Xác định hóa lập thể của 1-phenylpropane-1,2-diol trong phản ứng:

(A) (1R, 2R)


(B) (1S, 2S)
(C) (1R, 2S)
(D) (1S, 2R)
(E) Hỗn hợp đẳng lượng của (1R, 2R) và (1S, 2S)
(F) Hỗn hợp đẳng lượng của (1R, 2S) và (1S, 2R)
(G) Hỗn hợp đẳng lượng của (1R, 2R) và (1R, 2S)
(H) Hỗn hợp đẳng lượng của (1S, 2S) và (1S, 2R)
iii) Xác định công thức chất đầu A của phản ứng:

iv) Hãy đề xuất chuyển hóa trans-1,2-diphenylethylene (trans-stilbene)


thành meso- và d, l-1,2-diphenyl-ethanediol-1,2. Vẽ công thức chiếu
Fischer của các diol tạo thành (mạch carbon xếp dọc) và dẫn ra kí hiệu
R, S cho các tâm bất đối.

122
Hướng dẫn
i)

ii) Phản ứng oxi hóa (1E)-prop-1-en-1-ylbenzene bởi KMnO4 tạo thành hỗn
hợp đẳng mol các đồng phân (1R, 2R) và (1S, 2S) của 1-phenylpropane-1,2-
diol.

iii) A là 1,2-dimethylcyclopentene.

iv) Có thể sử dụng KMnO4 hoặc OsO4 để chuyển hóa convert trans-1,2-
diphenylethylene (trans-stilbene) thành meso-1,2-diphenylethanediol-1,2.
d,l-1,2-Diphenylethanediol-1,2 có thể được tạo thành bởi phản ứng của trans-
1,2-diphenylethylene với meta-chloroperbenzoic acid hoặc các peracid khác,
sau đó xử lí với dung dịch kiềm.

123
124
16. Phản ứng ozone phân và oxi hóa phân cắt
i) Xác định công thức alkene mà khi ozone phân sẽ tạo thành acetone và
methyl ethyl ketone.
ii) Hợp chất nào không được tạo thành trong phản ứng sau:

iii) Xác định công thức các sản phẩm chính A – H nhận được từ mỗi phản
ứng sau:

iv) Xác định sản phẩm chính của phản ứng sau:

125
v) Ozone phân hợp chất C6H12 chỉ tạo thành duy nhất một sản phẩm C3H6O.
Phản ứng bromo hóa gốc tự do C6H12 chỉ tạo thành duy nhất một sản
phẩm C6H11Br. Hãy xác định công thức chất C6H12 ban đầu và viết tất cả
các phương trình phản ứng.
Hướng dẫn
i) Hợp chất đó là 2,3-dimethylpent-2-ene:

ii) Phản ứng không tạo thành hợp chất D.

iii)

126
iv)

127
v) Hợp chất C6H12 ban đầu là 2,3-dimethylbut-2-ene (tetramethylethylene).

128
17. Phản ứng của epoxi
Dự đoán cấu trúc các hợp chất A, B tạo thành trong các chuyển hóa sau đây.
Với hợp chất B, hãy dẫn ra công thức chiếu Fischer và kí hiệu các tâm bất
đối theo hệ thống R, S. Cho biết B là đồng phân erythro- hay threo-?

Hướng dẫn
Tạo ra oxirane từ bromohydrin:

Mở vòng oxirane tạo thành các glycol:

129
18. Chuyển hóa hữu cơ
Sử dụng bất kì tác nhân cần thiết nào khác, hãy đề xuất cách chuyển hóa
methylenecyclopentane thành các hợp chất sau:

Hướng dẫn

130
19. Xác định hydrocarbon
Có 6 alkene vòng, không chứa nhóm ethyl, là đồng phân, có cùng công thức
C5H8. 3 trong số 6 hợp chất trên được cho vào các bình dán nhãn A, B, C
nhưng bạn không biết hợp chất trong mỗi bình là gì. Dựa vào kết quả của
các phản ứng với KMnO4 sau, hãy xác định công thức của các chất A-F.
- Hợp chất A tạo thành dicarboxylic acid D chứa một nguyên tử carbon thủ
tính.
- Hợp chất B tạo thành ketone E không chứa nguyên tử carbon thủ tính
nào.
- Hợp chất C tạo thành F chứa cả nhóm chức carboxylic acid và ketone và
cũng có một nguyên tử carbon thủ tính.
Hướng dẫn

131
20. Xác định hydrocarbon
Hydrocarbon C5H10 [A] tham gia vào các phản ứng sau:
+
A ⎯⎯⎯⎯
KMnO4 /H
→ Hai acid
A ⎯⎯⎯
Br2 /CCl4
→ B (dạng threo)
A ⎯⎯⎯⎯⎯1. OsO4
2. NaHSO3 /H2O
→ C (dạng erythro)

Xác định cấu trúc A, B, C và biểu diễn các cấu trúc của B và C theo công thức
chiếu Fischer. Viết các phương trình phản ứng.
Hướng dẫn

132
21. Xác định hydrocarbon
Hydrocarbon C8H16 [A] tham gia vào các phản ứng sau:
A ⎯⎯⎯
1.O3
2.Ph3P
→ B (ketone duy nhất)

A ⎯⎯→
Br2
CCl4
C (dạng meso)

A ⎯⎯⎯⎯1.BH3
2.H O ,OH−
→ D (racemate)
2 2

Xác định cấu trúc A - D và biểu diễn các cấu trúc của C và D theo công thức
chiếu Fischer. Viết các phương trình phản ứng.
Hướng dẫn

133
22. Sơ đồ chuyển hóa
1) Hoàn thành sơ đồ chuyển hóa sau:

2) Hydrocarbon C10H20 (A) tham gia vào các chuyển hóa sau:
A ⎯⎯⎯⎯⎯ 1.O3
2.Zn/CH3COOH
→ 2 ( CH3 )2 CHC ( O) CH3

A ⎯⎯⎯⎯
1.RCOOOH
2.H O/OH−
→ B (dạng meso)
2

A ⎯⎯
Br2
→ C (dạng meso)
A ⎯⎯⎯⎯1.BH3
2.H O /OH−
→ D (racemate)
2 2

Xác định cấu trúc A, B, C, D và biểu diễn các cấu trúc của B, C và D theo
công thức chiếu Fischer. Viết các phương trình phản ứng.

134
Hướng dẫn
1)

2)

135
136
23. Alkyne
i) Từ acetylene, 1-bromopropane và các tác nhân vô cơ cần thiết khác, hãy
đề xuất cách điều chế 1-bromopentane.
ii) Có thể nhận được tert-butyl acetylene bởi phản ứng giữa tert-butyl
bromide với sodium acetylenide không?
iii) Xác định các sản phẩm phản ứng a-k:

KAPA là muối potassium của propane-1,3-diamine.


iv) Đề xuất quy trình tổng hợp oxirane A từ ethylacetylene và 1-propane
iodine cùng các tác nhân vô cơ cần thiết khác. Gọi tên hợp chất A theo
danh pháp R, S.

137
Hướng dẫn
i) Giai đoạn đầu tiên là tạo thành anion acetylenide. Tiếp đó là phản ứng của
1-bromopropane với anion acetylenide, diễn ra theo cơ chế SN2:

ii) Không thể nhận được tert-butyl acetylene bởi sodium acetylenide sẽ đóng
vai trò như một base, gây ra phản ứng tách, tạo thành sản phẩm là
isobutylene.
iii) a) Phản ứng này là một phương pháp để điều chế các cis-alkene:

b) Phản ứng Kucherov, một phương pháp để tổng hợp các ketone:

c) Chuyển hóa hydrobor hóa – oxi hóa, một hướng để tạo ra các aldehyde:

d) Phản ứng này là phương pháp để tổng hợp các trans-alkene. Sự tạo thành
các trans-alkene được giải thích bởi trung gian anion-gốc trans-vinyl bền hơn.

e) Một phương pháp khác để điều chế các cis-alkene nhờ phản ứng cộng
hợp syn của borane:

138
f) Phản ứng này thể hiện sự ưu tiên vị trí của phản ứng cộng hợp
disiamylborane cồng kềnh (bị án ngữ không gian) vào liên kết ba:

g) Một ví dụ về tổng hợp các acetylene alcohol:

h) Phản ứng đồng phân hóa của một alkyne giữa mạch thành một alkyne đầu
mạch kém bền về mặt nhiệt động học hơn:

iv) A có thể được điều chế dưới dạng hỗn hợp các đối quang:

139
Quy trình:

140
24. Alkyne
i) Từ acetylene và 1-bromopropane, hãy đề xuất quy trình điều chế
pentanal và pentan-2-one với yêu cầu là trong cả hai trường hợp thì ở giai
đoạn cuối, chất đầu phản ứng phải giống nhau.
ii) Chọn ra các tác nhân phù hợp nhất dưới đây để chuyển hóa 3,3-
dimethylbut-1-yne thành 3,3-dimethylbutanal.
(A) Н2О, HgSO4 ( xúc tác ) , H2SO4
(B) 1. Sia2BH, THF, 0o C; 2. H2O2 , NaOH, H2O
(C) KMnO4 , NaOH, H2O
(D) 1. HOBr; 2. NaOH, H2O
iii) Hãy chọn ra điều kiện phù hợp nhất để chuyển hex-3-yne thành cis-hex-
3-ene.
(A) Xúc tác Pt catalyst, H2
(B) Xúc tác Lindlar, H2
(C) Na, NH3(l)
(D) NaNH2, NH3 (l)
iv) Khi xử lí với AgNO3 /NH3 / C2H5OH thì hydrocarbon C6H10 tạo thành một
muối bạc không tan. Oxi hóa hydrocarbon này bởi KMnO4 /H2O / to tạo
thành carboxylic C5H10O2. Phản ứng với H2O /HgSO4 /H2SO4 dẫn đến sự
tạo thành một ketone C6H12O. Xác định cấu tạo của hydrocarbon C6H10
này, biết rằng trong phổ 1H NMR cuả cả acid và ketone đều chỉ quan sát
thấy hai mũi đơn của các proton.
v) Hoàn thành sơ đồ chuyển hóa dưới đây:

141
vi) Hoàn thành sơ đồ tổng hợp dodeca-5,7-diyne B và hợp chất C.

vii) Đề xuất quy trình tổng hợp (5E, 7Z)-dodeca-5,7-diene từ hex-1-yne và các
tác nhân phổ biến sẵn có.
Hướng dẫn
i)

142
ii)

iii) Điều kiện phù hợp nhất là sử dụng H2 với xúc tác Lindlar (B).
iv) Hydrocarbon đó là 3,3-dimethylbut-1-yne.

v)

143
vi)

vii) Có thể thực hiện tổng hợp (5E, 7Z)-dodeca-5,7-diene từ hex-1-yne bởi các
phản ứng ghép cặp chéo.

144
145
25. Tổng hợp cyclopropane
Từ propylene, 1-bromopropane, CH2I2 và các tác nhân vô cơ cần thiết, hãy
đề xuất quy trình tổng hợp cis-1-methyl-2-propylcyclopropane.
Hướng dẫn

Phản ứng cyclopropane hóa cũng có thể được thực hiện bởi tác động của
CH2N2 / Pd (OAc )2 / Et2O .

146
26. Hệ liên hợp
i) Hợp chất nào sau đây có nhiệt hydrogen hóa cao hơn?

ii) Cấu trúc nào sau đây có chứa hệ liên hợp?

iii) Phản ứng cộng 1 đương lượng bromine vào hexa-2,4-diene ở 0 oC tạo
thành 4,5-dibromohex-2-ene (A) và đồng phân B. Đồng phân B là
(A) 5,5-Dibromhex-2-ene
(B) 2,5-Dibromhex-3-ene
(C) 2,2-Dibromhex-3-ene
(D) 2,3-Dibromhex-3-ene
iv) Sau phản ứng của khí HCl với 2-methylbuta-1,3-diene, phân lập được bốn
hợp chất mới (dữ kiện từ sắc kí khí-lỏng). Hãy xác định cấu tạo các hợp
chất này.
v) Xác định các hợp chất A – G:

vi) Xác định cấu tạo các sản phẩm chính của phản ứng cộng 1,2- và 1,4- của
khí hydrogen chloride vào 3-methylenecyclohexene và chỉ ra trong số
chúng sản phẩm nhiệt động học và sản phẩm động học.

147
vii) Giải thích sự tạo thành các hợp chất A – C trong phản ứng:

Hướng dẫn
i) Do trong A có chứa hệ liên hợp 1,3-diene nên chất này bền hơn, do đó chất
B (kém bền hơn) sẽ có nhiệt hydrogen hóa cao hơn.
ii) Các cấu trúc C và D có chứa hệ liên hợp. Trong cấu trúc A, nguyên tử
carbon mang điện tích dương và liên kết đôi bị tách ra bởi một nguyên tử
carbon lai hóa sp3, điện tích không thể được giải tỏa, do đó không có sự liên
hợp. Trong cấu trúc B, liên kết đôi và nhóm carbonyl cũng bị tách ra bởi một
nguyên tử carbon lai hóa sp3.
iii) 2,5-dibromohex-3-ene (B).
iv) Sơ đồ dưới đây giải thích cho sự tạo thành bốn sản phẩm mới A – D (dữ
kiện GLC) trong phản ứng của khí HCl với 2-methylbuta-1,3-diene. Sự proton
hóa của các liên kết đôi thứ nhất và thứ hai tạo thành các carbocation allyl
được bền hóa cộng hưởng. Do đó, sự tác kích của ion chloride dẫn đến sự
tạo thành bốn alkyl halide đồng phân, trong đó chiếm hàm lượng chủ yếu là
các đồng phân C và D.

148
v)

vi) Trước tiên, cần xác định được nguyên tử carbon lai hóa sp2 nào của hệ
liên hợp là carbon C1. Proton sẽ gắn vào nguyên tử đã được xác định, do
trong trường hợp này carbocation tạo thành bền hơn carbocation nhận được
khi proton gắn vào các phần khác của hệ liên hợp. Sản phẩm động học là 3-
methyl-3-chlorocyclohexene, trong khi đó 1-methyl-3-chlorocyclohexene là
sản phẩm nhiệt động học – bởi nó chứa liên kết đôi nhiều nhóm thế hơn.

vii) Giải thích tạo thành các hợp chất A – C trong phản ứng của HCl với 1-
methylcycloocta 1,3-diene:
i) Phản ứng cộng có thể diễn ra ở liên kết đôi C1-C2 nhiều nhóm thế
hơn, dẫn đến sự tạo thành một carbocation bậc ba. Nucleophile sẽ
gắn vào carbocation này, tạo thành sản phẩm cộng 1,2.

149
ii) Phản ứng cộng có thể diễn ra ở liên kết đôi C3-C4 ít nhóm thế hơn,
tạo thành một carbocation bậc hai. Nucleophile sẽ gắn vào
carbocation này, tạo thành sản phẩm cộng 1,2.

iii) Phản ứng cộng có thể diễn ra ở liên kết đôi C3-C4 ít nhóm thế hơn,
tạo thành một carbocation bậc hai. Dạng cộng hưởng của cation này
là một cation bậc ba dạng allyl bền hơn. Nucleophile sẽ gắn vào
carbocation này, tạo thành sản phẩm cộng 1,4.

150
27. Phản ứng Diels-Alder
i) Xác định sản phẩm tạo thành trong phản ứng sau, chỉ rõ hóa lập thể.

ii) Xác định các diene được dùng trong các phản ứng sau:

151
iii) Hoàn thành sơ đồ phản ứng:

Hướng dẫn
i) Phản ứng Diels-Alder có tính đồng bộ (toàn bộ các liên kết mới được tạo
thành đồng thời) và diễn ra với đặc trưng lập thể (cộng cis vào liên kết đôi)
– nghĩa là cấu hình của diene và dienophile được bảo toàn trong quá trình
tạo thành sản phẩm cộng. Trong tiến trình phản ứng, có hai liên kết σ và một
liên kết π mới được tạo thành.

Một cách viết khác:

152
ii)

a) b)

c) d)

e) f)

g) h)

iii) Chất A là benzo-1,4-quinone.

153
28. Phản ứng Diels-Alder
i) Có bao nhiêu cyclohexene đồng phân có thể được tạo thành trong phản
ứng Diels-Alder của diene A và dienophile B (không kể đến các đối
quang)?

ii) Xác định các sản phẩm A – D tạo thành bởi các phản ứng Diels-Alder:

154
Hướng dẫn
i)

ii)

155
29. Phản ứng vòng hóa Diels-Alder
1) Từ triphenylphosphine, ethyl bromoacetate, allyl bromide và
cyclopentene, cùng các tác nhân cần thiết khác, hãy điều chế hợp chất
A có công thức dưới đây.

2) Từ furan, acetylene, formaldehyde và buta-1,3-diene, cùng các tác nhân


cần thiết khác, hãy điều chế hợp chất B.

156
Hướng dẫn
1)

2)

157
30. Hiện tượng đồng phân lập thể của các hợp chất vòng
Một ví dụ về hiện tượng đồng phân là đồng phân cis-trans trong các hợp chất
vòng. Sự khám phá ra hiện tượng đồng phân này trong nhiều hợp chất tự
nhiên, dược phẩm và các hợp chất có hoạt tính sinh học khác đã thúc đẩy
sự phát triển của các phương pháp tổng hợp hữu cơ, dẫn đến việc tạo thành
những hợp chất với sự sắp xếp phù hợp của các nhóm thế với mặt phẳng
vòng. Hãy cùng xem xét các bài tập được mô tả dưới đây, nói về hiện tượng
đồng phân cis-trans, và trả lời các câu hỏi.

(Kí hiệu là A1, A2, … và B1, B2, … nếu có nhiều sản phẩm)
A. Hai phản ứng của 3-methylcyclohexane được biểu diễn trong sơ đồ trên.
1) Vẽ công thức cấu tạo của tất cả các sản phẩm tạo thành trong các phản
ứng. Lưu ý rằng phản ứng sử dụng hỗn hợp racemic của 3-
methylcyclohexane.
2) Xác định cấu hình tuyệt đối của tất cả các nguyên tử carbon bất đối hiện
diện trong sản phẩm.
3) Vẽ công thức dạng ghế của sản phẩm trong đó tất cả các nguyên tử
carbon bất đối có cấu hình R.
4) Vẽ công thức cấu tạo của các trung gian trong cả hai phản ứng. (Bỏ qua
hóa lập thể).
B. Các đồng phân hình học của 3,4-dimethylcyclopentanone, C và D (chú ý:
một trong chúng tồn tại ở dạng racemate) phản ứng với NaBH4 trong ethanol,
và sau đó được thủy phân trong môi trường acid. Trong phổ 1H NMR của các
sản phẩm tạo thành từ đồng phân C chỉ có một bộ tín hiệu, còn với các sản
phẩm từ D thì quan sát thấy hai hợp chất riêng biệt.
1) Vẽ công thức cấu tạo của C và D và các sản phẩm cuối tương ứng trong
các phản ứng.
2) Có bao nhiêu tín hiệu trong phổ 1H NMR của các hợp chất C và D? Đánh
dấu các nhóm proton trong công thức cấu tạo của C và D có cùng độ
chuyển dịch hóa học.

158
C. Các hợp chất đồng phân E và F (có công thức dưới đây), được tạo thành
trong phản ứng Diels-Alder. Vẽ công thức cấu tạo các hợp chất để tổng hợp
E và F.

Hướng dẫn
A.1-2)

3)

4)

B.1) C là hỗn hợp racemic.

159
Phản ứng khử các đồng phân tạo thành các alcohol sau: Phản ứng khử của
đồng phân D tạo thành hai hợp chất riêng biệt trong phổ 1H NMR.

Hai đồng phân dia


Đồng phân C tạo thành hỗn hợp hai đối quang, có phổ 1H NMR giống nhau.

Hai đối quang (không phân biệt được trong phổ 1H NMR)
2) Các đồng phân cis và trans, đều có 4 nhóm tín hiệu trong phổ 1H NMR.

C.
Tổng hợp E từ Tổng hợp F từ

160
31. Ảnh hưởng của tính thơm
1) Từ benzene, ethanol cùng các tác nhân cần thiết khác, hãy điều chế 1-
bromo-2,4,6-triethylbenzene. Viết sơ đồ phản ứng của chất này với:
a) potassium amide trong ammonia lỏng.
b) potassium amide trong ammonia lỏng khi có mặt kim loại
potassium.
2) Điều chế tetracene [A] từ benzene, 2,3-dimethylnaphthalene cùng các
tác nhân cần thiết khác.

3) Từ naphthalene, propanol-1, succinic anhydride cùng các tác nhân cần


thiết khác, hãy điều chế 9-(n-propyl)phenanthrene [B].

161
Hướng dẫn
1)

2)

162
3)

163
32. Cơ chế phản ứng
Đề xuất cơ chế cho phản ứng sau:

Từ ethylbenzene cùng các tác nhân cần thiết khác, hãy đề xuất hướng tổng
hợp A.
Hướng dẫn

Quy trình tổng hợp:

164
Chương 3

Dẫn xuất halogen, oxygen và


nitrogen
1. Tổng quan về phản ứng thế nucleophile
1) Xét cơ chế phản ứng của tert-butyl chloride với ion iodide:
(CH3 )3 C − Cl + I− → (CH3 )3 − I + Cl−

Nếu tăng gấp đôi nồng độ ion iodide thì tốc độ tạo thành tert-butyl iodide
sẽ
(A) tăng 2 lần.
(B) tăng 4 lần.
(C) giữ nguyên.
(D) giảm.
(E) không phải các phương án trên.
2) Hợp chất nào sau đây không tham gia vào các phản ứng thế theo cơ
chế SN1 và SN2?

3) Mỗi dẫn xuất halide sau có thể tham gia phản ứng thế nucleophile theo
cơ chế nào (SN1 hay SN2)? Sắp xếp các halide theo trình tự khả năng
tham gia phản ứng SN1.

165
4) Sắp xếp các chloride sau theo trình tự giảm dần hoạt tính dung môi phân
trong HCOOH 80%: diphenylchloromethane, tert-butyl chloride, benzyl
chloride, triphenylchloromethane.
5) Xác định công thức chất phản ứng (chất nền), tác nhân, dung môi và sản
phẩm trong tổng hợp 4-(cyanomethyl)cyclohexene bởi phản ứng thế
nucleophile.
6) Chất nào sau đây có hoạt tính kém nhất trong phản ứng SN2?

7) Sắp xếp các hợp chất sau theo thứ tự tăng dần khả năng phản ứng với
AgNO3. Giải thích.

Hướng dẫn
1) tert-Butyl chloride là dẫn xuất halide có nhiều nhóm thế cồng kềnh (quanh
nhóm chloride bị án ngữ không gian) nên có xu hướng dễ tham gia phản ứng
thế theo cơ chế SN1. Xét cơ chế SN1 cho phản ứng của tert-butyl chloride với
ion iodide thì thấy nucleophile không tham gia vào giai đoạn tốc định nên
nếu tăng gấp đôi nồng độ ion iodide thì tốc độ tạo thành tert-butyl iodide
được giữ nguyên (C).

166
2) Các hợp chất C và E không tham gia vào các phản ứng thế nucleophile
SN1 và SN2.
Các vinyl và aryl halide không tham gia phản ứng SN2 là do sự tiếp cận của
nucleophile từ hướng ngược lại bị ngăn cản bởi đám mây electron π của liên
kết đôi và vòng benzene.
Khi halogen liên kết với nhóm vinyl hoặc aryl thì trên nguyên tử carbon lai
hóa sp2 có một điện tích dương riêng phần và liên kết C-Hal bền hơn; ngoài
ra, các cation vinyl và aryl kém bền hơn các alkyl carbocation, khiến cho
phản ứng theo cơ chế SN1 không thuận lợi và gần như không thể diễn ra.
3) Các allyl và benzyl chloride đều có thể phản ứng theo cả cơ chế SN1 (tạo
ra carbocation bền) lẫn SN2 (trạng thái chuyển tiếp được bền hóa bởi một hệ
liên hợp).
Độ bền tương đối của các carbocation tương ứng với dẫn xuất halide ban
đầu giảm theo trình tự F  G  D  E  A  J  C  H  I  B

4) Phản ứng dung môi phân của các chloride trong HCOOH 80% diễn ra theo
cơ chế SN1. Cation triphenylmethyl là trung gian bền nhất nên chloride tương
ứng có hoạt tính mạnh nhất.

5)

167
6) Hợp chất B do trường hợp này có sự án ngữ không gian lớn nhất đối với
việc tiếp cận của nucleophile với tâm phản ứng (tâm electrophile).
7) Phản ứng của các chlorohydrocarbon với dung dịch AgNO3 tạo thành
alcohol xảy ra theo cơ chế SN1. Giai đoạn quyết định tốc độ phản ứng SN1 là
sự hình thành carbocation, do đó hợp chất nào tạo thành carbocation bền
nhất thì phản ứng nhanh nhất.
- Khả năng phản ứng của A kém nhất vì carbocation không có cấu trúc
phẳng (do tồn tại cấu trúc cầu nối.) Ngoài ra, cation này cũng không
được bền hoá bởi sự giải toả điện tích (cộng hưởng) làm tăng sức căng
của hệ quá nhiều.
- Hợp chất C dễ phản ứng nhất do tạo thành carbocation bậc III, có thể tồn
tại cấu trúc phẳng, và được bền hoá bởi sự cộng hưởng từ ba vòng thơm.
Do vậy: A < B < C.

168
2. Tính nucleophile
i) Lấy ví dụ về các nucleophile chứa С-, N-, O-, S-.
ii) Hãy dẫn ra định nghĩa về tính base và tính nucleophile. Trong một chu kì
và một nhóm của bảng tuần hoàn, tính base và nucleophile của các hợp
chất tương ứng biến đổi như thế nào?
iii) Độ mạnh-yếu của nucleophile phụ thuộc vào một vài yếu tố. Hãy cho biết
phát biểu nào sau đây không đúng:
(A) Anion là nucleophile mạnh hơn nguyên tử trung hòa hoặc phân tử.
(B) Các nguyên tử càng phân cực mạnh thì tính nucleophile càng mạnh.
(C) Mức độ solvate hóa ảnh hưởng đến tính nucleophile.
(D) Trong một chu kì của Bảng tuần hoàn, tính nucleophile và tính base
của các hợp chất tương ứng biến đổi cùng chiều.
(E) Trong một nhóm của Bảng tuần hoàn, tính nucleophile và tính base
của các hợp chất tương ứng biến đổi cùng chiều.
iv) Sắp xếp các anion và hợp chất trung hòa sau theo trình tự giảm dần tính
nucleophile trong dung dịch nước.

v) Sắp xếp các anion và hợp chất trung hòa sau theo trình tự giảm dần tính
nucleophile.
CF3CH2O− CH3CH2O− CH3COO− CH3CH2S− (CH3CH2 )2 O CH3CH2Li

vi) Phân loại các anion và phân tử trung hòa sau thành các nhóm theo độ
mạnh tương đối của tính nucleophile so với methanol.
RCOOH,Br − ,HS− ,H2O,RO− ,RCOO− ,OH− ,I− ,NH3 ,Cl− ,ROH,F − ,NC− ,RS− ,N3−
Rất tốt
Tốt
Chấp nhận được
Yếu
Rất yếu
vii) Tính nucleophile của các tiểu phân trong dãy I-, Br-, Cl-, F- biến đổi như
thế nào từ các dung môi aprotic sang dung môi protic?

169
Hướng dẫn
i)

ii) Tính base là khả năng phản ứng của một phân tử hoặc ion với proton hoặc
các Brønsted acid, còn tính nucleophile là khả năng phản ứng với carbon có
tính electrophile, nghĩa là với carbocation (sp2) hoặc với nguyên tử carbon
lai hóa sp3 mang điện tích dương riêng phần. Trong một nhóm của bảng tuần
hoàn, theo chiều từ trên xuống, tính nucleophile của các hợp chất tương ứng
sẽ tăng, trong khi đó tính base sẽ giảm (với các phản ứng trong dung môi
protic). Còn trong một chu kì, tính nucleophile và tính base đều tăng theo
chiều từ phải sang trái.
iii) Phát biểu e sai. Trong một nhóm của Bảng tuần hoàn, tính nucleophile
và tính base của các hợp chất tương ứng biến đổi ngược chiều.
iv) Nucleophile yếu
nhất là methanol
trung hòa. Các
nucleophile chứa
oxygen được sắp xếp theo độ mạnh của các acid liên hợp (pKa: HOH (15.7),
phenol (10), acetic acid (4.76)). Do lưu huỳnh có kích thước lớn hơn oxygen
nên CH3S- > HO-.
v) CH3CH2Li  CH3CH2S−  CH3CH2O−  CF3CH2O−  CH3COO−  (CH3CH2 )2 O

vi)
Rất tốt I− ,HS− ,RS−
Tốt Br − ,HO− ,RO− ,NC− ,N3−
Chấp nhận được NH3 ,Cl− ,F − ,RCOO−
Yếu H2O,ROH
Rất yếu RCOOH

170
vii) Trong các dung môi aprotic (ví dụ như dimethylformamide, acetonitrile,
acetone, tetrahydrofuran, dimethyl sulfoxide, hexamethyl phosphotriamide)
thì tính nucleophile biến đổi như sau: I- < Br- < Cl- < F-. Trong các dung môi
này, các nucleophile không bị solvate hóa mạnh như trong các dung môi
protic. Sự tiêu hao năng lượng để khử-solvate hóa biến đổi trong khoảng
hẹp. Hoạt tính của các nucleophile chủ yếu được quyết định năng lượng thu
được khi hình thành liên kết giữa nucleophile và nguyên tử carbon của chất
phản ứng, nghĩa là ái lực với tâm electrophile này – cũng giống như ái lực
đối với proton, đạt giá trị lớn nhất trong trường hợp fluorine.
Trong các dung môi protic (ví dụ như nước, formic acid, methanol, ethanol,
acetic acid): I- > Br- > Cl- > F-. Trong các dung môi protic, tác nhân nucleophile
được solvate hóa mạnh hơn nhiều so với trong các dung môi aprotic, do các
liên kết hydrogen xuất hiện giữa nucleophile và dung môi. Sự tiêu hao năng
lượng để khử-solvate hóa của nucleophile có thể so sánh với năng lượng thu
được từ việc tạo thành liên kết halogen-carbon mới và phụ thuộc vào khả
năng phân cực (bán kính) của ion halide.
Do đó, trong các dung môi aprotic, tính nucleophile của ion halide tỉ lệ thuận
với tính base của chúng, còn trong dung môi protic thì chủ yếu được xác định
bởi khả năng phân cực của chúng.

171
3. Ảnh hưởng của dung môi
i) Phát biểu nào sau đây về ảnh hưởng của dung môi trong các phản ứng
thế nucleophile là không chính xác:
(A) Các dung môi phân cực với hằng số điện môi lớn tạo thuận lợi cho
phản ứng SN1.
(B) Các dung môi aprotic phân cực tạo thuận lợi cho phản ứng SN2.
(C) Các alkyl halide vẫn có thể tham gia phản ứng thế SN2 trong các dung
môi phân cực với hằng số điện môi lớn.
(D) Các alkyl halide bậc ba vẫn có thể tham gia phản ứng SN1 trong các
dung môi không phân cực.
(E) Sự chọn lọc lập thể của các phản ứng với sự tham gia của alkyl halide
bậc hai không thể bị tác động bởi dung môi.
ii) Giải thích xu hướng biến đổi giá trị tốc độ tương đối của phản ứng giữa
n-butyl bromide với sodium azide trong các dung môi khác nhau.
CH3CH2CH2CH2Br + NaN3 → CH3CH2CH2CH2N3 + NaBr
Dung môi CH3OH H 2O DMSO DMFA CH3CN [(CH3)2N]3P=O
Tốc độ tương đối 1 7 1300 2800 5000 200000

Cho biết:

iii) Giải thích ảnh hưởng của dung môi đến các phản ứng bậc hai sau:
a) Tốc độ phản ứng tương đối trong H2O / trong CH3OH / trong C2H5OH
= 1/16/44.
131 −
I + CH3I → CH3131I + I−
b) Tốc độ phản ứng tương đối trong hexane / trong chloroform =
1/13000.
(CH3CH2CH2 )3 N + CH3I → (CH3CH2CH2 )3 N+ (CH3 )I−

172
Hướng dẫn
i) Đánh giá các phát biểu:
- (A) đúng. Các phản ứng SN1 cần các dung môi phân li proton với hằng
số điện môi ε lớn. Vai trò của các dung môi như vậy (H2O, HCOOH,
CH3OH, CH3COOH) là để làm bền hóa các cation và anion.
- (B) đúng. Với các phản ứng SN2, không thực sự cần dung môi protic, do
chỉ có cation (ion bù trừ của nucleophile) cần được solvate hóa. Các
dung môi tốt có thể kể đến là: acetone, các ether (diethyl ether, dioxane,
tetrahydrofuran), acetonitrile, dimethylformamide, dimethyl sulfoxide,
hexamethylphosphotriamide.
- (C) và (D) đúng, phản ứng vẫn có thể xảy ra nhưng với tốc độ rất chậm.
- (E) sai vì sự chọn lọc lập thể của các phản ứng thế của alkyl halide bậc
hai có thể bị ảnh hưởng bởi dung môi.
ii) Do n-butyl bromide là alkyl halide bậc một nên phản ứng thế ưu tiên diễn
ra theo kiểu SN2. Các dung môi protic (CH3OH, H2O) làm chậm tốc độ phản
ứng, trong khi đó các dung môi aprotic phân cực (DMSO, DMF, CH3CN,
[(CH3)2N]3P=O) thì phù hợp với phản ứng SN2. Các dung môi protic làm chậm
tốc độ phản ứng SN2 bởi chúng solvate hóa nucleophile, làm giảm năng
lượng và hoạt tính của nó. Các dung môi aprotic phân cực thì hòa tan được
nhiều muối nhưng chúng solvate các cation thay vì anion có tính nucleophile.
Kết quả là các anion không bị solvate hóa có thêm hoạt tính nucleophile và
làm tăng tốc độ phản ứng SN2.
iii) a) Dung môi với độ phân cực kém hơn (ethanol kém phân cực hơn nước)
sẽ thuận lợi hơn.

b) Trong trường hợp này, trạng thái chuyển tiếp phân cực hơn, do đó dung
môi phân cực hơn (chloroform) sẽ thuận lợi hơn.

173
4. Ảnh hưởng của nhóm rời đi
i) Cho biết mối liên hệ giữa tính base và khả năng đóng vai trò như một
nhóm rời đi:
(A) Không có liên hệ gì cả.
(B) Nhóm rời đi tốt là các base mạnh.
(C) Nhóm rời đi tốt là các base yếu.
ii) Sắp xếp các nhóm rời đi (được in đậm) theo mức độ dễ dàng thay thế
trong các phản ứng thế nucleophile:

iii) Sắp xếp các nhóm rời đi dưới đây vào bảng mức độ dễ dàng tách chúng
khỏi nguyên tử carbon của chất nền trong phản ứng thế nucleophile:

TsO− , Br − , H2O, OH− , I− , NH2− , Cl− , RO− , F− , NR3

Cực kì tốt
Rất tốt
Tốt
Chấp nhận được
Kém
Rất kém
iv) Ethyl halide nào sau đây tham gia phản ứng SN2 nhanh nhất?

v) Sắp xếp các hợp chất dưới đây theo trình tự hoạt tính trong các phản
ứng SN2.

vi) Alkyl bromide nào sau đây có hoạt tính mạnh nhất trong phản ứng SN1?

174
vii) Carbocation nào sau đây có năng lượng cao nhất?

Hướng dẫn
i) Các nhóm rời đi tốt là những base yếu của các acid liên hợp mạnh (C).
ii) Một nhóm rời đi tốt nên là base liên hợp của acid mạnh.

iii)
Cực kì tốt TsO− , NR3
Rất tốt I− ,Br −
Tốt H2O
Chấp nhận được Cl−
Kém F−
Rất kém OH− , NH2− , RO−
iv) Dãy hoạt tính của các ethyl halide trong phản ứng SN2: D > C > A > B.
Trong phản ứng SN2, nucleophile tác kích vào nguyên tử carbon của chất nền
từ phía ngược lại với nhóm rời đi. Theo khả năng dễ tách ra, các ion halide
được sắp xếp như sau: I- > Br- > Cl- > F-. Dãy này tương ứng với sự tăng dần
khả năng phân cực của ion halide (từ F- đến I-), nghĩa là xu hướng tách ra sẽ

175
tăng lên theo “tính mềm” của halogen. 1-Iodoethane (D) phản ứng nhanh
nhất do ion iodide là nhóm rời đi tốt nhất trong dãy này.
v) Hợp chất hoạt động nhất là methyl bromide do đây là alkyl halide bậc một
với nhóm rời đi tốt (C > A > B > D).
vi) Trong các phản ứng SN1, alkyl bromide C có hoạt tính mạnh nhất. Điều
then chốt là phải biết phản ứng SN1 diễn ra như thế nào. Với phản ứng SN1,
việc tách nhóm rời đi diễn ra trước khi tạo thành liên kết với nucleophile. Do
đó, trung gian là carbocation. Hoạt tính của các alkyl halide R-Hal trong phản
ứng SN1 tăng nếu điện tích dương của carbocation R+, tạo thành trong giai
đoạn tốc định, được bền hóa (giải hỏa) bởi các hiệu ứng +I hoặc +M của các
nhóm thế. Trong trường hợp này, carbocation bậc ba bền nhất.
vii) Năng lượng càng cao thể hiện qua độ bền càng thấp. Độ bền phụ thuộc
vào việc liệu có các nhóm nhường hoặc nhận electron liên kết với nguyên tử
carbon dương điện không. Trong số các carbocation đã cho, carbocation
bậc một (A) có năng lượng cao nhất – vì xung quanh không có nhóm nhường
electron nào để giải tỏa điện tích.

176
5. Hóa lập thể
i) Các phản ứng SN1 và SN2:
(A) đều đặc trưng lập thể.
(B) đều chọn lọc lập thể.
(C) SN1 không đặc trưng lập thể, SN2 thì có.
(D) SN1 đặc trưng lập thể, SN2 thì không.
(E) đều không đặc trưng lập thể lẫn chọn lọc lập thể.
ii) Biểu diễn cấu trúc các chất nền và sản phẩm thủy phân (R)-2-bromoctane
trong một phản ứng SN1 lí tưởng.
iii) Xác định sản phẩm của việc đun sôi (S)-3-chlorocyclopent-1-ene trong
methanol.
iv) Phát biểu nào sau đây về sự chuyển vị carbocation không chính xác:
(A) Có thể diễn ra trong các phản ứng SN1.
(B) Có thể diễn ra trong các phản ứng SN2.
(C) Đôi khi bao gồm sự chuyển dịch nguyên tử hydrogen.
(D) Diễn ra do độ bền cao hơn của các carbocation bậc hai.
(E) Diễn ra nhanh hơn phản ứng giữa các phân tử.
Hướng dẫn
i) SN1 không đặc trưng lập thể, SN2 thì có (C).
ii)

iii) Việc đun hồi lưu (S)-3-chlorocyclopent-1-ene trong methanol tạo ra điều
kiện phù hợp với phản ứng SN1. Nucleophile tác kích vào carbocation phẳng
dẫn đến sự tạo thành hỗn hợp racemic của các đồng phân ether.

iv) Ý sai là: Có thể diễn ra trong các phản ứng SN2 (B).

177
6. Sản phẩm phản ứng thế
i) Thủy phân 2-bromo-3-methylbutane (I) và 3-bromo-2,2-dimethylbutane
(II) trong điều kiện của các cơ chế phản ứng SN1 và SN2 lần lượt dẫn đến
sự tạo thành các đồng phân A, B (từ I) và C, D (từ II). Xác định điều kiện
của mỗi phản ứng, biểu diễn cấu tạo và đề xuất cơ chế tạo thành các hợp
chất A, B và C, D.

ii) Đề xuất cơ chế phản ứng để giải thích các kết quả dưới đây:

iii) Giải thích hoạt tính tương đối của các hợp chất sau trong phản ứng thủy
phân trong dung dịch ethanol.

iv) Xác định sản phẩm thủy phân 1-bromobut-2-ene trong các điều kiện
phản ứng SN1 và SN2.
v) Giải thích các dữ kiện thực nghiệm sau:

178
vi) Chất nào sau đây có hoạt tính kém nhất trong phản ứng SN2?

vii) Sắp xếp các alkyl bromide dưới đây theo trình tự tăng dần hoạt tính trong
các phản ứng SN2.

(CH3 )3 CCH2 − Br (CH3 )2 CH − Br (CH3 )3 C − Br CH3CH2 − Br

179
Hướng dẫn
i) Tùy thuộc vào các điều kiện thủy phân mà 2-bromo-3-methylbutane (I) và
3-bromo-2,2-dimethylbutane (II) sẽ phản ứng theo cơ chế SN1 và SN2, lần lượt
tạo thành các đồng phân A, B và C, D. Các phản ứng của nucleophile yếu
(H2O) diễn ra theo cơ chế SN1, trong đó có sự chuyển vị của các carbocation.
Còn khi sử dụng nucleophile mạnh (NaOH) thì diễn ra phản ứng SN2.

ii) Phản ứng SN2 của alkyl halide được tạo thuận lợi bởi nồng độ nucleophile
mạnh trong dung môi aprotic phân cực. Dưới các điều kiện phản ứng SN2
(MeONa/DMF, nucleophile mạnh + dung môi aprotic), khi nguyên tử bromine
bị thay thế bởi nhóm methoxy, cấu hình được bảo toàn với sự tạo thành chỉ
hợp chất B.

180
Phản ứng SN1 của alkyl halide được tạo thuận lợi bởi các tính chất
nucleophile yếu của methanol và khả năng ion hóa được tăng cường (cũng
nhờ nhiệt độ cao) của nó trong vai trò dung môi. Dưới các điều kiện của phản
ứng SN1 (MeOH/ 100 oC, nucleophile yếu + dung môi protic), nucleophile
(methanol) tấn công vào nguyên tử carbon của carbocation phẳng với xác
suất ở hai phía bằng nhau, dẫn đến sự tạo thành các lượng đẳng mol của
các đối quang B và C.

iii) Hoạt tính tương đối cho thấy các bromide bậc ba A - C liên quan đến thực
tế rằng theo điều kiện đã cho (thủy phân trong dung dịch ethanol), phản ứng
diễn ra theo cơ chế SN1 qua carbocation phẳng. Không thể nhận được cation
phẳng từ các hợp chất B và C (dạng hình học cực kì không thuận lợi, bị giới
hạn bởi các vòng cứng nhắc), do đó, các cation tương ứng rất kém bền và
phản ứng diễn ra chậm hơn nhiều.
iv)

v) Sơ đồ tổng quát cho phản ứng SN1 có thể được biểu diễn như sau:

181
Nucleophile có thể tác kích bất kì dạng nào của vi hạt ion. Nếu nucleophile
tác kích carbocation đã phân li hoàn toàn, sản phẩm sẽ được racemic hóa
hoàn toàn. Mặt khác, nếu nucleophile tác kích hoặc cặp ion tiếp xúc, hoặc
cặp ion đã tách ra-được solvate hóa, thì nhóm rời đi sẽ gây cản trở một phần
sự tiếp xúc của nucleophile từ phía này của carbocation và do đó sẽ nhận
được nhiều hơn sản phẩm nghịch đảo cấu hình (chú ý rằng nếu nucleophile
tác kích vào phân tử chưa phân li thì sản phẩm sẽ có cấu hình nghịch đảo).

vi) Do sự án ngữ không gian nên hợp chất B có hoạt tính trong phản ứng SN2
kém nhất.
vii) Yếu tố không gian đóng vai trò cực kì quan trọng.

182
7. Sản phẩm phản ứng thế
i) Dự đoán những sản phẩm nào có thể được tạo thành khi dung môi phân
neopentyl chloride trong HCOOH 80%.
ii) Hãy dự đoán alkyl halide nào sau đây sẽ có hiệu suất cao nhất trong
phản ứng thế với CH3CH2ONa tạo thành ether:

iii) Ether nào không thể được tạo thành bởi phản ứng Williamson?

Hướng dẫn
i) Trong quá trình dung môi phân neopentyl chloride trong HCOOH 80%, phản
ứng diễn ra theo cơ chế SN1. Carbocation tạo thành ở giai đoạn đầu tiên trải
qua chuyển vị Wagner – Meerwein. Carbocation bậc ba mới phản ứng với
nước tạo thành một alcohol bậc ba. Sự tách proton tạo thành các alkene.

183
ii) Halide A (1-bromopentane) có thể tạo thành ether (sản phẩm thế trong
phản ứng Williamson) với hiệu suất tốt nhất. Các điều kiện phản ứng tương
ứng với cơ chế SN2. Với phản ứng SN2, sự tác kích của nucleophile phải diễn
ra theo hướng ngược lại của nhóm rời đi. Do đó, nguyên tử carbon không bị
án ngữ không gian sẽ bị tác kích. Trong trường hợp có sự án ngữ không gian,
ví dụ như bromide bậc ba C (1-bromo-2,2-dimethylpropane) phản ứng với
CH3CH2ONa thì phản ứng tách chiếm ưu thế.

iii) Diphenyl ether (C) không thể được tạo thành bởi phản ứng Williamson,
do các vinyl và aryl halide không trải qua phản ứng thế nucleophile. Có thể
tổng hợp ester này dưới các điều kiện khác (ví dụ, khi có mặt muối đồng).

184
8. Phản ứng thế nucleophile
Hãy đề xuất chuyển hóa (R)-butanol-2 thành các hợp chất sau:
(A) (S)-2-fluorobutane
(B) (R)-butanethiol-2
(C) (R, S)-sec-butylethyl ether
Hướng dẫn

185
9. So sánh tốc độ phản ứng
So sánh tốc độ phản ứng trong các dung môi khác nhau:
a) Dung môi: C2H5OH (1) và DMSO (2).

b) Dung môi: H2O (1) và hỗn hợp H2O/acetone (2).

c) Dung môi: CH3OH (1) và DMFA (2):

Chỉ ra kiểu cơ chế của các phản ứng trên.


Hướng dẫn
a) Cơ chế E2, tốc độ phản ứng trong DMSO (2) cao hơn trong C2H5OH (1).
b) Cơ chế SN1, tốc độ phản ứng trong H2O (1) cao hơn trong hỗn hợp
H2O/acetone (2).
c) Cơ chế SN2, tốc độ phản ứng trong DMFA (2) cao hơn trong CH3OH (1).

186
10. So sánh tốc độ phản ứng
1) Sắp xếp các hợp chất sau theo trình tự giảm dần tốc độ dung môi phân
trong formic acid:

2) Sắp xếp các hợp chất RI theo trình tự giảm dần hoạt tính trong phản ứng
sau: RCl + KI ⎯⎯⎯
acetone
→RI + KCl , với R = n-C3H7, sec-C4H9, iso-C4H9, CH3,
PhCH2, CH2=CH, CH3COCH2, CH=CHCH2.
3) Hợp chất A có công thức phân tử C3H7ClO, dễ bị dung môi phân trong
ethanol (0 oC, chu kì bán phản ứng 10 phút). Hợp chất B có cùng công
thức phân tử nhưng dưới cùng điều kiện như trên thì phản ứng chậm hơn
10 lần so với hợp chất A. Hợp chất C, có công thức phân tử C3H7Cl, phản
ứng với sodium phenoate trong ethyl alcohol ở 80 oC tạo thành một sản
phẩm thế với tốc độ chậm hơn 10 lần so với ethyl chloride. Hãy đề xuất
cấu tạo của các hợp chất A, B, C.
4) So sánh tốc độ dung môi phân trong formic acid của các cặp chất sau:

5) Sắp xếp các hợp chất RX theo trình tự giảm dần hoạt tính trong phản
ứng sau: RX + KCN ⎯⎯⎯
DMSO
to
→RCN + KX

187
Hướng dẫn
1) Cơ chế SN1:
(p-CH3O-C6H4)2CHCl ≫ (C6H5)3CCl > (C6H5)2CHCl > C6H5CH2Cl > (CH3)3CCl
> CH3(C2H5)CHCl > CH3CH2CH2CH2Cl
2) Cơ chế SN2:
CH3COCH2Cl > C6H5CH2Cl > CH2=CHCH2Cl > CH3Cl > CH3CH2CH2Cl >
(CH3)2CHCH2Cl > CH3CH2CH(CH3)Cl ≫ CH2=CHCl
3) A: CH3CH(OH)CH2Cl; B: HOCH2CH2CH2Cl; C: (CH3)2CHCl
4)

5) [5] > [2] > [1] > [4] > [6] > [3] > [7]

188
11. Cơ chế phản ứng
1) Thủy phân allyl halide bậc ba A, có công thức phân tử C6H11Cl, và đồng
phân allyl halide bậc một B của nó thì tạo thành hỗn hợp hai alcohol đồng
phân C và D với tỉ lệ bằng nhau. Xác định công thức cấu tạo của A – D
và viết phương trình phản ứng thủy phân, cho biết phản ứng thuộc loại
cơ chế nào.
2) Xác định các sản phẩm (ít nhất ba) của phản ứng acetol phân (dung môi
phân trong acetic acid) của cyclohexene-4-ol tosylate.
3) Phản ứng dưới đây khá thú vị khi xét trên phương diện cơ chế.

a) Dự đoán phản ứng đã xảy ra như thế nào bằng cách viết ra cơ chế
từng giai đoạn.
b) Dự đoán sản phẩm tạo thành khi thay chất đầu bằng 1,4-
dimethylbenzene.

189
Hướng dẫn
1)
Phản ứng diễn ra theo cơ chế SN1:

190
2)
Có thể tạo thành các sản phẩm I, II, III, IV, trong đó I và II là sản phẩm chính.

3) a) Trong giai đoạn đầu tiên, tert-butyl alcohol bị chuyển thành carbocation
tương ứng. Tiểu phân này có thể mất 1 proton, tạo thành 2-methylpropene;
hoặc lấy 1 hydride ion từ nhóm isopropyl của chất đầu.
- Hướng phản ứng thứ hai khả thi, bởi carbocation mới bền hơn tert-
butylcarbocation do nó cũng là carbocation bậc ba, hơn nữa còn được
bền hoá bởi hiệu ứng cộng hưởng của vòng thơm.

- Carbocation mới này phản ứng với 2-methylpropene giàu electron tạo thành
một tiểu phân trung gian carbocation khác. Tiểu phân này có phản ứng thế
electronphil nội phân tử với vòng thơm, tạo thành sản phẩm cuối.

191
b) Với 1,4-dimethylbenzene thì chỉ tạo thành 2,5-dimethyl-tert-butylbenzene
qua một phản ứng thế electrophile thông thường do sự tách hydride ion từ
nhóm methyl không ưu tiên xảy ra.

192
12. Cơ chế phản ứng
Đưa ra lời giải thích ngắn gọn cho các quan sát sau đây:
a) Trong những điều kiện giống nhau, phản ứng của NaSCH3 với 1-bromo-
2-methylbutane chậm hơn phản ứng với 1-bromobutane khá nhiều.
b) Khi xử lý (S)-2-butanol tinh khiết quang học phản ứng với một base mạnh
như LiNH2, rồi phục hồi, thì tính quang hoạt vẫn giữ nguyên. Nhưng nếu
cho (S)-2-butanol phản ứng với nước ấm, có một lượng nhỏ sulfuric acid,
thì sau khi phục hồi, alcohol bị mất tính quang hoạt.
c) Phản ứng của cyclobutene với bromine (Br2, lạnh, trong bóng tối) tạo
thành một hỗn hợp racemic. Trong khi đó, phản ứng với hydrogen “nặng”
khi có xúc tác platinum (D2/Pt) lại tạo thành một hợp chất meso.
d) Đun hồi lưu (R)-2-bromobutane với dung dịch NaOH đặc trong ethanol,
thu được (S)-2-butanol.
e) Đun hồi lưu (R)-2-bromobutane với dung dịch NaOH loãng trong ethanol
thu được hỗn hợp racemic của 2-butanol. Tốc độ tạo thành alcohol thay
đổi thế nào khi: i. tăng gấp đôi nồng độ alkyl bromide; và ii. tăng gấp đôi
nồng độ NaOH?
f) Trong các điều kiện giống nhau, phản ứng của các đồng phân dia (xuyên
lập thể phân) A, B tạo thành các sản phẩm khác nhau rõ rệt. Gợi ý: Hãy
chú ý đến khía cạnh lập thể không gian ba chiều.

193
Hướng dẫn
a) Cả hai phản ứng đều thuộc loại SN2, xảy ra ở nguyên tử carbon bậc một.
Phản ứng SN2 chịu ảnh hưởng chính bởi các tương tác không gian. 1-bromo-
2-methylbutane phân nhánh ở vị trí , gây ra sự cản trở không gian khi tác
nhân nucleophile thiolate tấn công. Do vậy phản ứng của nó chậm hơn 1-
bromobutane.
b) Phản ứng đầu tiên chỉ có sự deproton hoá/proton hoá và chỉ ảnh hưởng
tới liên kết O-H chứ không làm thay đổi cấu hình của tâm chiral. Do vậy nó
không bị mất tính quang hoạt. Phản ứng thứ hai có sự tham gia của liên kết
C-O của tâm chiral và có sự hình thành một tiểu phân carbocation với cấu
trúc phẳng. Khi cộng hợp nước vào carbocation phẳng này thì phản ứng có
thể xảy ra ở cả hai phía, dẫn tới sự hình thành một hỗn hợp racemic và làm
mất tính quang hoạt.

c) Phản ứng cộng bromine là quá trình anti (trái phía), dẫn tới sự tạo thành
một sản phẩm trans. Sự tấn công của ion Br+ vào hai mặt (trên và dưới) của
cyclobutene đều tạo ra cùng một tiểu phân bromonium ion. Tiếp đó, bromide
ion tấn công, mở vòng tiểu phân này, tạo thành một hỗn hợp sản phẩm (R,R)-
and (S,S)- với số mol bằng nhau (racemate), hỗn hợp này không có tính
quang hoạt.
Phản ứng hydrogen hoá lại là quá trình syn (cùng phía), dẫn tới sự tạo thành
một sản phẩm cis. Khả năng cộng hợp vào hai phía của cyclobutene là tương
đương nhau, nhưng lần này sản phẩm tạo thành lại chỉ là một hợp chất meso.
Sản phẩm này cũng không có tính quang hoạt.

194
d) Tác nhân nucleophile (OH-) có nồng độ cao thì phản ứng ưu tiên xảy ra
theo cơ chế SN2, dẫn tới sự nghịch đảo cấu hình 100 % ở vị trí xảy ra sự thế.
e) Nồng độ nucleophile thấp thì phản ứng ưu tiên xảy ra theo cơ chế SN1, đi
qua trạng thái carbocation. Tiểu phân trung gian phẳng này có thể “bắt” dung
môi hoặc tác nhân nucleophile ở cả hai phía, dẫn tới sự racemic hoá. Trong
các điều kiện đã cho, tốc độ phản ứng SN1 chỉ phụ thuộc vào nồng độ alkyl
halide, do vậy khi tăng gấp đôi nồng độ alkyl halide thì tốc độ tạo ra alcohol
tăng gấp đôi Còn tăng gấp đôi nồng độ nucleophile (OH-) không làm ảnh
hưởng đến tốc độ phản ứng.
f) Phản ứng (1) xảy ra theo cơ chế SN2, trong đó tác nhân nucleophile (ion
CH3O-) thay thế cho bromide anion, dẫn tới sự nghịch đảo cấu hình. Phản
ứng tách không thể xảy ra bởi không tồn tại cấu dạng nào có nguyên tử Br
và nguyên tử hydrogen của carbon liền kề đều ở vị trí liên kết trục (sự sắp
xếp anti-periplanar).
Phản ứng (2) xảy ra theo cơ chế tách E2. Phản ứng này dễ xảy ra bởi sự “lật
vòng” tạo thành cấu dạng với sự sắp xếp anti-periplanar của hydrogen và
bromine.

195
13. Hóa lập thể của phản ứng thế
A. Phản ứng của (R)-2-hydroxysuccinic acid với phosphorus trichloride tạo
thành sản phẩm B.

1) Xác định cấu trúc (R)-2-hydroxysuccinic acid.


2) Xác định cấu trúc trung gian A và sản phẩm B, chỉ rõ hóa lập thể.
3) Đề xuất cấu trúc trạng thái chuyển tiếp trong các chuyển hóa của A, B.
B. Phản ứng của (1S, 2S)-trans-p-bromobenzenesulfonate của
iodocyclohexyl (W) với acetic acid tạo thành trans-acetate của 2-
iodocyclohexyl (T).
1) Viết cấu dạng ghế của W(e,e) và W(a,a). Với e là equatorial (biên), còn a
là axial (trục).
Phản ứng diễn ra theo sơ đồ dưới đây:

2) Xác định các trung gian X+, Y+ và T trong giản đồ trên và viết cấu dạng
của chúng.
3) Xác định cấu hình R/S của các nguyên tử carbon thủ tính trong T.

196
Hướng dẫn
A.1)

2)

3)

B.1)

197
2)

198
14. Xác định hợp chất hữu cơ
Hợp chất A (C5H8O, đối quang S tinh khiết), có thể được chuyển thành hợp
chất B (C5H7Br, đối quang R tinh khiết) qua một quá trình hai giai đoạn:
i) CH3SO2Cl, triethylamine
ii) LiBr
Khi phản ứng với khi hydrogen, có xúc tác kim loại, B chuyển thành hai phân
tử phi thủ tính C, D (đều có công thức phân tử C5H9Br). Chuyển B thành hợp
chất Grignard tương ứng, rồi thuỷ phân với nước, thu được hợp chất E (C5H8,
phi thủ tính). Cho E phản ứng với dung dịch acid của KMnO4, tạo thành F
(C5H8O3). Phổ hồng ngoại của F cho thấy có sự tồn tại của 2 nhóm carbonyl
khác nhau và 1 nhóm hydroxy. Xác định cấu trúc hoá lập thể của các hợp
chất A-F.
Hướng dẫn

199
15. Chuyển hóa hữu cơ
1) Tiến hành chuỗi chuyển hóa như sau với (R)-1-phenylpropanol-2 chứa
đồng vị đánh dấu 18O:

Xác định cấu trúc các chất A, B, chỉ rõ cấu hình tâm thủ tính. Chất nào
trong các sản phẩm cuối có chứa đồng vị đánh dấu?
2) So sánh góc quay của mặt phẳng ánh sáng phân cực đối với ether B,
được tạo thành theo hai cách khác nhau từ alcohol quang hoạt:

3) Hoàn thành chuỗi chuyển hóa sau và chỉ ra cấu hình tuyệt đối (R/S) của
các hợp chất nhận được ở mỗi giai đoạn:

4) Hoàn thành chuỗi chuyển hóa sau và chỉ ra cấu hình tuyệt đối (R/S) của
các hợp chất nhận được ở mỗi giai đoạn:

200
Hướng dẫn
1)

2)

3)

Cơ chế phản ứng thế với KOH/H2O-EtOH (bảo toàn cấu hình):

4)

201
16. Chuyển hóa hữu cơ
1) Hoàn thành chuỗi chuyển hóa sau:

2) Hoàn thành chuỗi chuyển hóa sau:

3) Hoàn thành chuỗi chuyển hóa dưới đây:

Hướng dẫn
1)

202
2)

Hiệu suất của D cao hơn đáng kể so với G do cấu dạng I, trước khi diễn ra
phản ứng tách E2, thuận lợi về mặt năng lượng hơn cấu dạng II – với các
nhóm phenyl cồng kềnh ở gần nhau.

3)

203
17. Ảnh hưởng của nhóm kề
1) Hoàn thành chuỗi chuyển hóa sau. Đề xuất cơ chế khả dĩ cho bước cuối
cùng.

2) Trình bày cơ chế đồng phân hóa các p-haloamine:

Tại sao cân bằng chiếm ưu thế bởi halide chứa nguyên tử halogen ở carbon
bậc hai? Tại sao phản ứng thủy phân các β-chloroamine R2NCH2CH(Cl)CH3
trong các môi trường kiềm và acid lại tạo thành các β-aminoalcohol đồng
phân?

3) Tốc độ dung môi phân trong acetic acid (acetol phân) khi có mặt sodium
acetate của các đồng phân hình học của 2-acetoxycyclohexyl tosylate
có sự khác biệt rõ rệt: đồng phân trans phản ứng nhanh hơn 670 lần so
với đồng phân cis. Ngoài ra, trong cả hai trường hợp, diacetate tạo thành
đều có cấu hình trans.

204
Phản ứng acetol phân dưới các điều kiện này của trans-2-
acetoxycyclohexyl tosylate quang hoạt tạo thành racemic của trans-
diacetate. Hãy giải thích:
(1) Sự khác biệt rõ ràng giữa tốc độ acetol phân của các đồng phân cis
và trans tosylate.
(2) Kết quả hóa lập thể quan sát được từ phản ứng acetol phân của các
đồng phân này.
(3) Sự tạo thành trans-diacetate trong phản ứng acetol phân của trans-
2-acetoxycyclohexyl tosylate quang hoạt.
4) Giải thích sự khác nhau về kết quả hóa lập thể trong phản ứng của methyl
ester của (S)-2-bromopropionic acid với sodium methoxide và muối
sodium của (S)-2-bromopropionic acid với methanol.

Việc thêm muối bạc vào sẽ ảnh hưởng gì đến tốc độ phản ứng thứ hai?
Điều này có làm thay đổi kết quả hóa lập thể không?

205
Hướng dẫn
1)

2) Cơ chế phản ứng đồng phân hóa:

Trong môi trường acid mạnh, nguyên tử nitrogen bị proton hóa hoàn toàn và
cặp electron chưa liên kết của nó không tham gia vào quá trình thủy phân
dẫn xuất halogen. Do đó, phản ứng diễn ra như một phản ứng thế bình
thường, nghĩa là không có sự tham gia của nhóm kề.
Còn nếu amine không bị chuyển thành dạng muối thì giai đoạn đầu tiên của
quá trình thủy phân là phản ứng thế SN2 nội phân tử, tạo thành ion immonium
A. Khi có mặt base mạnh, ion này tiếp tục trải qua phản ứng mở vòng theo
cơ chế SN2 ở nguyên tử carbon bậc một hoạt tính mạnh hơn, tạo thành sản
phẩm chuyển vị.

206
3) Sự khác biệt rõ rệt về tốc độ acetol phân được giải thích bởi sự tham gia
của nguyên tử oxygen trong hợp phần carbonyl (nhóm carboxyl) trong trường
hợp đồng phân trans, còn trong trường hợp đồng phân cis thì không có sự
tham gia của nhóm kề.

4) Kết quả hóa lập thể của phản ứng thứ hai sẽ không bị ảnh hưởng khi thêm
muối bạc, nhưng tốc độ phản ứng thì sẽ tăng lên, bởi ion bạc sẽ tạo phối trí
với bromine, giúp tăng tốc độ tách loại ion bromide này.

207
18. Cơ chế phản ứng
Đề xuất cơ chế cho các phản ứng sau:

Hướng dẫn
a)

b)

208
c)

d)

e)

f)

209
19. Cơ chế phản ứng
Đề xuất cơ chế các phản ứng sau:

210
Hướng dẫn
a)

b)

c)

d)

211
19A. Cơ chế phản ứng
Từ khung bixiclo[4.1.0]heptane có thể mở rộng vòng để tạo thành hệ vòng
7 cạnh.
a) Cho biết cấu trúc của A, B trong hai phản ứng dưới đây

b) Giải thích sự tạo thành một trong hai sản phẩm dựa trên cấu dạng của
chất đầu.

Hướng dẫn
Cấu trúc các sản phẩm

Do vòng ba cạnh phẳng nên nó buộc các nguyên tử carbon số 1, 2, 3 và 4


cũng phải phẳng (tương tự vòng cyclohexane). Như vậy cấu dạng chính xác
phải là dạng bán ghế thay vì dạng ghế thông thường,

Carbocation không cổ điển


Giai đoạn nước cộng vào carbocation không cổ điển có thể được biểu diễn
chính xác hơn như sau:

212
20. Cấu dạng và phản ứng tách
Sự chuyển vị của các phân tử hoạt tính là loại phản ứng hữu cơ cực kì quan
trọng (đặc biệt trong các phản ứng xúc tác bởi enzyme). Việc nghiên cứu
thường tập trung vào thành phần hoạt tính của phân tử, kiểu và tốc độ phản
ứng.
1) So sánh các phân tử benzene và cyclohexane (hình 1, hình 2): chỉ ra trạng
thái lai hóa của mỗi nguyên tử carbon và góc gần đúng giữa hai liên kết
C-C cạnh nhau trong mỗi phân tử. Các phân tử này có phẳng hay không.

Cấu dạng (sự sắp xếp không gian) bền nhất của cyclohexane là cấu dạng
ghế. Có hai cấu dạng như vậy và chúng tồn tại trong một cân bằng (hình 3).
Ở cấu dạng trái, các nguyên tử HA và HD ở vị trí trục, còn các nguyên tử HB và
HC ở vị trí biên. Sự nghịch đảo vòng diễn ra làm vị trí của các nguyên tử
hydrogen thay đổi. Các cyclohexane có nhóm thế duy trì cấu dạng mà các
nhóm thế lớn ở vị trí biên, bởi cấu dạng này bền về mặt năng lượng hơn.
Trong tất cả các câu hỏi tiếp theo, hãy vẽ tất cả các cấu dạng theo cách như
trong hình 3.
2) Vẽ các cấu dạng ghế cho mỗi hợp chất dưới đây. Chỉ ra đâu là cấu dạng
bền hơn.

3) Vẽ cấu dạng ghế bền nhất của cis-decalin và trans-decalin (bao gồm cả
các nguyên tử hydrogen được dẫn ra) và chỉ ra hợp chất nào, cis-decalin
hay trans-decalin, bền hơn?

213
4) Xác định sản phẩm của các phản ứng tách E2 sau đây. Đề xuất cơ chế
của các phản ứng (sử dụng các cấu dạng ghế). Một trong các phản ứng
không diễn ra, hãy cho biết đó là phản ứng nào và giải thích tại sao.

Hướng dẫn
1) Benzene: các nguyên tử carbon đều lai hóa sp2, góc giữa các liên kết C-C
là 120o, phân tử phẳng.
Cyclohexane: các nguyên tử carbon đều lai hóa sp3, góc giữa các liên kết C-
C là 109.5o, phân tử không phẳng.
2) a) Hai cấu dạng như nhau, không thể so sánh tính bền:

b) Cấu dạng bên phải bền hơn:

c) Cấu dạng bên trái bền hơn:

214
3)

Trans-decalin bền hơn bởi tất cả các nhóm thế alkyl đều ở vị trí biên, trong
khi đó cis-decalin có một nhóm thế alkyl ở vị trí trục và một nhóm biên.
4) Để phản ứng tách E2 diễn ra, cả nguyên tử hydrogen bị base tác kích lẫn
nhóm rời đi phải ở trên các nguyên tử carbon cạnh nhau và phải ở vị trí trục.
Phản ứng (c) không diễn ra bởi nguyên tử bromine và nhóm tert-butyl cồng
kềnh phải ở vị trí trục, rất không thuận lợi về mặt năng lượng.

215
21. Tổng hợp thyroxine
Thyroxine (K) là một hormone tiết ra từ tuyến giáp.

Đi từ tyrosine (А), hợp chất K được tổng hợp theo sơ đồ sau:

Phản ứng của tyrosine (А) với nitric acid được tiến hành trong các điều kiện
như vậy sẽ diễn ra sự thế hai lần. Hợp chất B tạo thành ở dạng ion lưỡng
cực. B phản ứng với acetic anhydride theo tỉ lệ mol 1:1 tạo thành C. Hợp chất
C có kết quả dương tính với thuốc thử FeCl3. Sự khử F thành G diễn ra với
lượng dư hydrogen. Các chuyển hóa G → H và I → J cũng sử dụng tác nhân
dư. Hợp chất J phản ứng với I2 theo tỉ lệ mol 1:2.
1) Xác định cấu hình tuyệt đối (R/S) cho các tâm lập thể của hợp chất A
theo quy tắc Cahn-Ingold-Prelog.
2) Hoàn thành tất cả các phản ứng trong sơ đồ trên, sử dụng cấu trúc ba
chiều để biểu diễn các chất B-K. Đưa ra kí hiệu dạng cơ chế (SN, E, …)
cho các chuyển hóa sau: A → B, B → C, C → D, E → F.
3) Đề xuất cơ chế chi tiết cho các chuyển hóa: a) A → B (giai đoạn nitro
hóa đầu tiên); b) B → C, c) C → D.
4) Xác định cấu hình tuyệt đối (R/S) cho các tâm lập thể của hợp chất K
theo quy tắc Cahn-Ingold-Prelog.

216
Hướng dẫn
1) Cấu hình S.
2)

Cơ chế phản ứng nitro hóa là SEAr.

Cơ chế phản ứng là AАс2.

Cơ chế phản ứng là ААс2.

217
Cơ chế phản ứng là SNAr.

Cơ chế phản ứng là SEAr.

218
3) a)

Phản ứng thế thứ hai cũng tương tự.


b)

219
c)

d) Hợp chất K có cấu hình S.

220
22. Tổng hợp amine
1) Đề xuất phương pháp tổng hợp tert-butylamine, đi từ tert-butanol, cùng
các tác nhân cần thiết khác.
2) Từ benzene và các tác nhân cần thiết khác, hãy tổng hợp 1,3-
dibenzoylbenzene.
3) Từ p-toluidine (4-methylaniline) và các tác nhân cần thiết khác, hãy tổng
hợp 2-hydroxy-4', 5-dimethyl-2', 3-dinitroazobenzene (A).

4) Hãy điều chế 2-hydroxy-3-fluorobenzaldehyde từ o-nitroaniline và các


tác nhân cần thiết khác.
5) Từ cyclohexanone và các tác nhân cần thiết khác, hãy tổng hợp amine
bậc ba A có cấu trúc như sau:

221
Hướng dẫn
1)

2)

3)

222
4)

5)

223
23. Xác định amine
Amine C7H13N (A) tạo thành phức chất với B(CH3)3 mạnh hơn phức của
triethylamine và không tạo thành sản phẩm bền trong phản ứng với
benzenesulfonyl chloride trong kiềm. Khi được đưa vào chuỗi thoái phân sau
đây thì tạo thành trivinylmethane.

Xác định cấu trúc các hợp chất A, B, C.


Hướng dẫn
Theo đề bài đã cho, amine C7H13N (A), phải là một amine bậc ba với cấu trúc
cứng nhắc, được cố định bởi hệ vòng bicyclic, trong đó cặp electron chưa
liên kết (trong orbital lai hóa sp3) không bị án ngữ bởi bộ khung carbon. A
phải là amine bậc ba bởi nó không tạo thành sản phẩm bền với C6H5SO2Cl
trong dung dịch kiềm (sản phẩm cộng này bị phân cắt bởi dung dịch NaOH,
hoặc thậm chí là chỉ bởi nước). Cấu trúc của A có thể được suy luận từ ba
giai đoạn liên tiếp của phản ứng tách muối ammonium hydroxide bậc bốn
theo Hofmann:

224
24. Xác định amine
Thiết lập cấu trúc của aminoalcohol có công thức C8H15NO (A), chứa một
nhóm hydroxyl bậc hai, dựa vào các thông tin từ chuyển hóa sau:

Biết muối D chứa một cation thơm, bền và không quang hoạt.
Hướng dẫn
Sản phẩm cuối của chuỗi thoái phân có chứa cation thơm, bền (C7H7+) –
nhiều khả năng là cation tropylium, do đó D là tropylidene (cycloheptatriene-
1,3,5), còn C là cycloheptadiene với một nhóm dimethylamino, bị tách ra
trong thoái phân Hofmann.

225
Chương 4

Hợp chất carbonyl và dẫn xuất


1. Phản ứng của các aldehyde và ketone
Cho sơ đồ phản ứng sau:

a) Xác định cấu trúc của A, B và các tác nhân c, d.


b) Phản ứng tạo thành A từ benzaldehyde thuộc loại nào? Trình bày cơ chế
phản ứng.
Các alcohol cũng phản ứng với aldehyde và ketone. Trước tiên, tạo thành
các hemiacetal, rồi đến acetal:
+ +
Aldehyde / ketone + alcohol ⎯⎯
H
→hemiacetal ⎯⎯
H
→acetal
Phản ứng tạo thành một acetal diễn ra qua nhiều giai đoạn:

c) Xác định cấu trúc của K-N.


d) Làm thế nào để cân bằng chuyển dịch theo chiều tạo thành acetal?

226
Các alcohol với nhiều hơn một nhóm -OH như glycerol (propane-1,2,3-triol)
cũng phản ứng với ketone tạo thành acetal.
e) Viết phương trình phản ứng của acetone và glycerol.
Xét các phản ứng sau của benzaldehyde:

f) Xác định cáu trúc O-T.


Phản ứng cuối ở ý e khi benzaldehyde tạo thành S và T là một ví dụ của phản
ứng Cannizzaro. Trong phản ứng này, carbon tự oxi hóa-khử (dị phân). Các
aldehyde không có α hydrogen thì có thể phản ứng theo cách này. Giai đoạn
đầu tiên của phản ứng Cannizzaro tạo thành một trung gian tứ diện.
g) Trình bày cơ chế phản ứng Cannizzaro, lấy ví dụ phản ứng của
benzaldehyde.
h) Giải thích tại sao các hợp chất có α hydrogen thì không xảy ra phản ứng
Cannizzaro.

227
Hướng dẫn
a)

b) Phản ứng thế nucleophile.

c)

d) Có thể dùng thiết bị tách nước để chuyển cân bằng theo chiều tạo thành
sản phẩm.
e)

228
f)

g)

h) Các aldehyde với hydrogen ở vị trí alpha sẽ tạo thành enol khi xử lí với
base. Do đó không thể diễn ra phản ứng Cannizzaro.

229
2. Các hợp chất carbonyl ở dạng chất phản ứng và sản
phẩm
Các hợp chất carbonyl là những nguồn giá trị đối với tổng hợp hữu cơ bởi
chúng phản ứng được với nhiều hợp chất khác nhau, thường thì có độ chọn
lọc. Trong sơ đồ phản ứng dưới đây, một số phản ứng đi từ butyl alcohol (1)
đã được mô tả:

a) Vẽ các cấu trúc A đến G.


b) Đưa ra một phương án khả thi để i) tổng hợp trực tiếp B từ butyl alcohol.
Hợp chất F là một epoxide.
c) Xác định tác nhân ii) cần để tổng hợp diol 2 và tác nhân iii) để chuyển
hóa diol 2 thành A. Gọi tên phản ứng của diol 2 thành A. (Gợi ý: Hợp chất
iii) có chứa một nguyên tử halogen ở số oxi hóa cao nhất).
d) Dự đoán sản phẩm từ phản ứng của hợp chất C với MeLi. Vẽ cấu trúc
sản phẩm.
e) Tại sao không thể tạo ra hợp chất D bằng cách cho B phản ứng trực tiếp
với MeLi?
Cuối cùng hợp chất D được chuyển thành 5.
f) Đề xuất các điều kiện phản ứng iv) cho phản ứng của D thành 5.

230
g) Trình bày cơ chế phản ứng này và cho biết chất trung gian và sản phẩm
thuộc nhóm hợp chất nào?
Hướng dẫn
a)

b) i: oxi hóa với K2Cr2O7, H2SO4 loãng (phản ứng oxi hóa Jones), hoặc với
CrO3, H2SO4 loãng, acetone hoặc với KMnO4
c) ii: Acid (H3O+) hoặc base (OH-) để mở vòng epoxide.
iii: Phân cắt periodate với NaIO4.
d)

e) MeLi chỉ có phản ứng dữ đội đi kèm với sự deproton hóa acid.
f) iv: Ethane diol, H+
g)

231
3. Nhận biết hợp chất hữu cơ
Các hợp chất sau chứa trong 5 bình mất nhãn. Một loạt các thí nghiệm xác
định nhóm chức đã được thực hiện với các chất có trong 5 bình. Dựa vào
các kết quả thực nghiệm, hãy xác định các chất sau có trong bình nào.

i) Lấy 5 ống nghiệm nhỏ, cho vào mỗi ống 1 mL acetone. Thêm khoảng
10 – 20 mg mỗi chất cần xác định vào mỗi ống nghiệm, tiếp đó nhỏ
một giọt thuốc thử chromic-sulfuric acid rồi lắc nhẹ để trộn lẫn các
chất với nhau. Sau vài phút, mẫu chứa A và C phản ứng với dung dịch
dichromate tạo thành dung dịch màu xanh ngọc lục bảo và một kết
quả có thể nhìn thấy. Để xác nhận màu xanh ngọc lục bảo của kết
tủa, phần váng nổi lên được gạn cẩn thận rồi cho vào ống nghiệm.
Kết tủa được rửa hai lần cho đến khi xuất hiện màu sắc rõ ràng.
ii) Lặp lại thí nghiệm (i), nhưng thay thuốc thử chromic-sulfuric acid
bằng một giọt dung dịch KMnO4 0.2 %, thì lại xảy ra sự chuyển màu
và chỉ quan sát được có kết tủa tạo thành trong các mẫu chứa A, C.
iii) Khi thêm dung dịch sodium hydroxide loãng vào các ống nghiệm rồi
lắc nhẹ để trộn lẫn các chất thì chỉ có mẫu chứa B tan. B cũng là hợp
chất duy nhất có phản ứng dương tính với giấy quỳ.
iv) Khi thêm dung dịch sodium hypoiodite (được điều chế bằng cách
hoà tan iodine vào dung dịch sodium hydroxide) vào các ống nghiệm
thì chỉ mẫu chứa A và E tạo ra kết tủa vàng tươi.
v) Các hợp chất C, D, E phản ứng với thuốc thử 2,4-dinitrophenyl
hydrazine (2,4-DNPH) tạo ra kết tủa đỏ cam.

232
Hướng dẫn
i) Phản ứng oxi hóa bởi chromate để xác định những nhóm chức dễ bị oxi
hóa như alcohol, aldehyde. Do vậy, chất 4, 5 phải là A, C (mặc dù chưa biết
chính xác từng chất.)
ii) Phản ứng oxi hóa bởi permanganate cũng để xác định những nhóm chức
dễ bị oxi hóa. Thí nghiệm này chỉ đơn thuần là củng cố lại những nhận định
từ i.
iii) Tan trong base và dương tính với giấy quỳ là tính chất của nhóm acid. Do
vậy B phải là chất có nhóm chức carboxylic acid – 2.
iv) Phản ứng với iodoform là của methyl ketone và methyl alcohol bậc II (chất
này bị thuốc thử oxi hóa thành methyl ketone), do vậy A, E phải là 3, 4 . Kết
hợp với dữ kiện ở i, ii ta xác định được các cặp chất: C-5; A-4; E-3. Chỉ còn
lại D, nó chỉ có thể là 1.
v) Phản ứng với 2,4-dinitrophenylhydrazine để xác định nhóm chức aldehyde
và ketone. Chỉ có 1, 3, 5 có phản ứng này. Kết quả này xác nhận những kết
luận trên là hợp lí.

233
3. Tổng hợp hữu cơ
Từ methyl ethyl ketone và propionaldehyde, cùng các tác nhân cần thiết
khác, hãy điều chế:
- 2,3-dimethylpent-2-enal [A]
- 2,3-dimethylpentanal [B]
- 2,3-dimethylpent-2-en-1-ol [C]
Hướng dẫn
Trước tiên, để điều chế aldehyde không no α, β A thì cần thực hiện phản ứng
ngưng tụ aldol-croton trực tiếp giữa các hợp chất carbonyl khác nhau:
propionaldehyde (hợp phần methylene) và methyl ethyl ketone (hợp phần
carbonyl). Sau đó, khử hóa chọn lọc liên kết đôi (để điều chế B) hoặc nhóm
carbonyl (điều chế C).

Liên kết đôi carbon-carbon trong aldehyde không no α, β A cũng có thể được
khử chọn lọc bởi phản ứng hydrogen hóa xúc tác ở 20 oC và áp suất thông
thường trên xúc tác palladium, ruthenium, hoặc platinum (sản phẩm là
aldehyde B). Sự khử hóa nhóm carbonyl của các α, β-enone và enal đạt được
khi sử dụng tác nhân khử diisobutylaluminium hydride (DIBAL-H) trong
toluene dưới các điều kiện rất êm dịu (sản phẩm là alcohol C).

234
3A. Điều chế nhóm chức ester
Nhóm chức ester khá phổ biến trong các hợp chất hữu cơ, và các nhà hoá
học cũng đã tìm ra nhiều phương pháp để tổng hợp nhóm chức quan trọng
này. Nhiều trong số đó có cơ chế tương tự nhau và đều liên quan đến sự thế
nucleophile của acyl.

Ngoài ra cũng có một số hướng tổng hợp khác. Dưới đây là hai phản ứng có
cơ chế tươg tự nhau. Hãy trình bày cơ chế của hai phản ứng này:

235
Hướng dẫn
Cả hai phản ứng đều có giai đoạn chuyển carboxylic acid thành muối
carboxylat (có tính nucleophile mạnh hơn), sau đó chất này tham gia phản
ứng thế SN2, tạo thành ester. Trong trường hợp đầu tiên, iodide anion là
nhóm rời đi. Còn ở trường hợp sau thì carboxylate (tạo thành khi proton
chuyển qua diazomethane) thay thế cho N2.

236
4. Các enol và enolate trong hóa hữu cơ
Sơ đồ 1 biểu diễn một hiện tượng mà một mặt là cơ sơ của các phản ứng
hữu cơ quan trọng, nhưng mặt khác lại thường gây ra những phản ứng phụ
không mong muốn. Dưới đây là một số câu hỏi liên quan đến hiện tượng này.

Sơ đồ 1
1. Xử lí isopropyl phenyl ketone với LDA (Lithium diisopropylamide), sau đó
alkyl hóa với dimethyl sulphate (xem sơ đồ 2). Kết quả của phản ứng này là
hai sản phẩm A và B được tạo thành. Tỉ lệ của các sản phẩm có thể thay đổi
bằng cách điều chỉnh điều kiện phản ứng. Các dung môi aprotic, phân cực
(ví dụ như HMPA, hexamethylphosphoramide) có thể làm tăng tỉ lệ sản phẩm
B. Dữ kiện 1H NMR của hợp chất A: 5H, mũi đa, 7 ppm; 3H, mũi đơn, 3.3 ppm;
3H, mũi đơn, 1.8 ppm; 3H, mũi đơn, 1.7 ppm. Dữ kiện 1H NMR của hợp chất
B: 5H, mũi đa, 7 ppm và 9H, mũi đơn, 1.2 ppm.

Sơ đồ 2
2. Ketone không no α, được biểu diễn trong sơ đồ 3, trải qua phản ứng cộng
1,2 hoặc 1,4 (cộng Michael) tùy vào nucleophile được sử dụng. Phản ứng với
tác nhân Grignard tạo thành hợp chất C, trong khi đó phản ứng với thiophenol
tạo thành hợp chất D.

Sơ đồ 3

237
3. Xử lí cyclohexanone và methyl vinyl ketone với base thì xảy ra phản ứng
vòng hóa Robinson (sơ đồ 4). Đấy là một quá trình gồm ba giai đoạn. Trước
tiên là phản ứng cộng Michael đã được đề cập ở trên, sau đó trung gian E
tạo thành trải qua phản ứng aldol hóa nội phân tử, tạo thành hợp chất F, cuối
cùng chất này trải qua phản ứng tách tạo thành sản phẩm vòng hóa Robinson
(G) với công thức C10H14O. Các dữ kiện phổ hồng ngoại và 1H NMR quan
trọng nhất của hợp chất G: Quan sát được trong phổ hồng ngoại một dải
mạnh ở 1705 cm-1; trong phổ 1H NMR, quan sát được một số tín hiệu đặc
trưng của các proton liên kết với các nguyên tử carbon lai hóa sp3 và một tín
hiệu đặc trưng của proton liên kết với nguyên tử carbon lai hóa sp2. (Chú ý:
Việc giải thích dữ liệu phổ đóng vai trò củng cố lập luận và không đòi hỏi
phải có trong lời giải cho bài này.)

a) Cho biết tên gọi hiện tượng trong Sơ đồ 1.


b) Xác định cấu trúc A, B.
c) Giải thích hiệu ứng dung môi đến sự phân bố sản phẩm alkyl hóa được
mô tả ở ý 1.
d) Trong câu: “LDA là base Brönsted mạnh/yếu và là nucleophile
mạnh/yếu” - hãy gạch bỏ hai từ tạo ra một phát biểu đúng. Giải thích
ngắn gọn.
e) Sử dụng thuyết acid/base cứng/mềm (HASB) để giải thích cấu trúc của
các sản phẩm C, D được tạo thành.
f) Xác định cấu trúc của các trung gian C, D và sản phẩm vòng hóa G.
Hướng dẫn
a) Hiện tượng hỗ biến (tautomer hóa) keto-enol.
b)

238
c) Trong trường hợp này, phản ứng O-alkyl hóa chiếm ưu thế, khi enolate bị
phân li (nhờ động lực từ HMPA, một dung môi aprotic phân cực), nó có thể
tạo phức với các lithium ion. Còn tert-butyl alcohol có thể tạo thành các liên
kết hydrogen với nguyên tử oxygen trong enolate, làm cản trở phản ứng O-
alkyl hóa và ưu tiên phản ứng C-alkyl hóa.
d) LDA là base mạnh, bởi nó là base liên hợp của acid yếu
(diisopropylamine), nhưng lại là tác nhân nucleophile yếu vì sự án ngữ không
gian gây ra bởi các nhóm thế isopropyl.
“LDA là base Brönsted mạnh/yếu và là nucleophile mạnh/yếu”.

e) Các tác nhân Grignard là các nucleophile cứng, theo thuyết HSAB thì nó
ưu tiên tấn công các tâm electrophile cứng, như là nguyên tử carbonyl
carbon. Còn thiophneol là nucleophile mềm, ưu tiên phản ứng với các tâm
electrophile như là nguyên tử vinyl carbon (beta).

f)

239
240
4A. Phản ứng ngưng tụ
Cho 1 mol semicarbazide (A) phản ứng với 1 mol cyclohexanone (B) và 1
mol furtural (C) trong ethanol có một lượng nhỏ acid (dạng vết) thì thu được
một hỗn hợp semicarbazone (D, E). Nếu kết thúc phản ứng sau 5 phút thì
hỗn hợp chủ yếu chứa semicarbazone D. Tuy nhiên, nếu thực hiện phản ứng
qua đêm, thì trong sản phẩm gần như chỉ có semicarbazone E. Sử dụng giản
đồ năng lượng để giải thích cho các kết quả này.

Hướng dẫn
Nhóm carbonyl trong cyclohexanone hoạt động hoá học (tính electrophile)
hơn trong furtural do furtural được bền hoá bởi hiệu ứng liên hợp với vòng
thơm. Do đó, nếu phản ứng bị ngừng lại sau một thời gian ngắn thì sản phẩm
chính là sản phẩm được tạo thành nhanh hơn (sản phẩm khống chế động
học – D). Tuy nhiên, phản ứng này xảy ra trong cân bằng và nếu thời gian đủ
lâu, sản phẩm chính là sản phẩm bền về mặt nhiệt động học hơn – E.

241
5. Chuỗi chuyển hóa
1) Hoàn thành chuỗi chuyển hóa sau đây:

2) Hoàn thành chuỗi chuyển hóa sau đây:

3) Hoàn thành chuỗi chuyển hóa sau đây:

4) Hoàn thành chuỗi chuyển hóa sau đây:

242
Hướng dẫn
1)

2)

243
3)

4)

Từ sơ đồ trên có thể thấy rằng trong tổng hợp các hợp chất carbonyl thế ở vị
trí α thì các enamine đóng vai trò là chất tổng hợp thay thế cho các anion
enolate, và các phản ứng β-alkyl hóa, β-acyl hóa, và cộng hợp Michael (phản
ứng Stork) diễn ra khi có sự tham gia của các enamine trong điều kiện trung
hóa, điều này làm giảm tối đa các quá trình ngưng tụ phụ.

244
6. Xác định hợp chất tự nhiên
Dựa vào các thông tin sau, hãy xác định cấu trúc hợp chất tự nhiên X, chất
này tham gia vào việc điều hòa nhiều quá trình sinh lí liên quan đến huyết áp
và các phản ứng dị ứng.
- Hợp chất X có thể được xem là heptanoic acid có nhóm thế với khối
lượng phân tử 354u và không chứa nguyên tử carbon bậc bốn.
- Hợp chất X không tạo thành phenylcarbazone.
- Phản ứng ozone phân của hợp chất X, được thực hiện trong môi trường
có tính oxi hóa, tạo thành ba hợp chất (A, B, C).
- Đốt cháy 1.32 g hợp chất X, tương ứng với 10 milimol thu được 2.2 g CO2
và 0.72 g H2O.
- Hợp chất X không có tâm lập thể và khi được đun nóng sẽ tạo thành
anhydride vòng D.
- Hợp chất B có thể được tạo thành ở dạng hỗn hợp racemic trong phản
ứng của n-hexanal với HCN, và sau đó thủy phân sản phẩm tạo thành Z,
quá trình được thực hiện trong môi trường acid.
- Cho hợp chất C phản ứng với CrO3, tạo thành hợp chất
bên.
- Cho hợp chất C phản ứng với 2 đương lượng acetyl
chloride thì thu được dẫn xuất F.
- Khử hợp chất C bởi lithium aluminium hydride thu
được hợp chất G.
a) Xác định cấu trúc của A.
b) Xác định cấu trúc hợp chất D.
c) Xác định cấu trúc các hợp chất Z và B.
d) Xác định các cấu trúc có thể có của hợp chất C và chọn ra cấu trúc phù
hợp chất với điều kiện đã cho trong đề bài.
e) Vẽ một trong các đồng phân lập thể có thể có của hợp chất X. Chỉ rõ các
tâm lập thể.
f) Dựa vào số lượng các tâm lập thể, hãy xác định số xuyên lập thể phân
(đồng phân dia) có thể có của hợp chất này.
g) Vẽ các đồng phân lập thể có thể có khác của hợp chất X (bỏ qua các
đồng phân quang học).
h) Xác định cấu trúc hợp chất F và G.

245
Hướng dẫn
a) Hợp chất A:

b) Hợp chất D:

c) Hợp chất Z và B lần lượt là:

d) Các cấu trúc có thể có của C:

Dựa vào thông tin ở ý thứ 7 thì không thể xác định cấu trúc của hợp chất C
bởi hợp chất này có 4 nhóm, mà một vài hoặc tất cả chúng có thể được tạo
thành bởi sự oxi hóa hợp chất C bởi CrO3. Thông tin ở ý thứ 2 cho phép loại
trừ các cấu trúc C2-C6. Vậy, cấu trúc duy nhất thỏa mãn đề bài là C1.
e)

246
f) 25 = 32 xuyên lập thể phân.
g) Đặt R= -CH2CH2COOH, và R’= -CH(OH)(CH2)4CH3

h) Cấu trúc F và G lần lượt là:

247
7. Ngưng tụ benzoin
2 mol benzaldehyde phản ứng được với 1 mol hợp chất A, sử dụng xúc tác
ion cyanide. A tồn tại ở dạng cặp đối quang.
Hợp chất A phản ứng với sodium borohydride tạo thành hợp chất B. B tồn tại
ở dạng 3 đồng phân thể.
a) Viết cơ chế phản ứng tạo thành A và phương trình phản ứng tạo thành B.
Xác định các cấu trúc của A và B.
b) Biểu diễn A, B ở dạng công thức tứ diện. Gắn các cấu hình R, S (theo quy
tắc Cahn-Ingold-Prelog) với mỗi chất.
Khi tiếp xúc với Al2O3, hợp chất B có phản ứng tách nước tạo thành C1 và
C2. C1 và C2 tồn tại trong một cân bằng với C3. Trong phổ IR của C1 và C2
có các dải hấp thụ quanh vùng 1600 cm-1, và cả C1 và C2 đều làm mất màu
nước bromine. Nếu xử lí B với sulfuric acid đặc để tách nước thì một phản
ứng hoàn toàn khác xảy ra. Hợp chất D được tạo thành với một dải hấp thụ
mạnh ở quanh vùng 1700 cm-1. Hợp chất D phản ứng với semicarbazide
(H2N-NH-CO-NH2) tạo thành một hợp chất tương đối kém tan.
c) Viết phương trình phản ứng tạo thành các hợp chất C1-C3 và D. Xác định
cấu trúc của chúng và viết cơ chế phản ứng tạo thành D.
Hợp chất D bị khử tạo thành hợp chất E.
d) Viết cấu trúc của hợp chất E.
Nếu xử lí E với acid mạnh thì phản ứng tách nước diễn ra tạo thành hợp chất
F. Ozone phân F rồi xử lí tiếp với tác nhân oxi hóa tạo thành sản phẩm G
(C13H10O).
e) Xác định cấu trúc của F và G.
f) Gọi tên G.

248
Hướng dẫn
a)

b)

249
c)

D là một aldehyde, thể hiện hấp thụ hồng ngoại đặc trưng ở khoảng 1700
cm-1. Nó tạo thành một semicarbazone kém tan.
d)

e-f)

250
8. Tổng hợp hữu cơ
1) Từ acetaldehyde, acetone, benzaldehyde và các tác nhân cần thiết khác,
hãy điều chế 3-hydroxy-3-methyl-5-phenylpentanal – một chất có mùi
hoa huệ thung lũng, được dùng làm nước hoa.
2) Đề xuất quy trình tổng hợp 5-hydroxypentanoic acid lactone từ
cyclohexanone và các tác nhân cần thiết khác.
3) Từ isobutyraldehyde (2-methylpropanal), piperidine và methyl vinyl
ketone, cùng các tác nhân cần thiết khác, hãy điều chế 4,4-
dimethylcyclohexen-2-one.
4) Điều chế hợp chất dưới đây từ bromobenzene, cyclohexanone và các tác
nhân cần thiết khác:

5) Điều chế oct-6-yne-2-one từ 5-hydroxypentan-2-one, methylacetylene và


các tác nhân cần thiết khác.
6) Từ diethyl malonate và ethylene oxide, cùng các tác nhân cần thiết khác,
hãy điều chế ester vòng  -butyrolactone của 4-hydroxybutanoic acid.
7) Từ acetone, ethyl acetoacetate và các tác nhân cần thiết khác, hãy điều
chế hợp chất A:

251
Hướng dẫn
1)

2)

3)

Phản ứng của enamine (hợp phần nhường, Michael donor) với methyl vinyl
ketone (hợp phần nhận, Michael acceptor) là một ví dụ điển hình của phản
ứng Michael.
4)

Phản ứng oxi hóa của các allyl và benzyl cũng có thể được tiến hành bởi tác
động của MnO2 trong methylene chloride.

252
5)

6)

7)

253
9. Tổng hợp hữu cơ
1) Thực hiện chuyển hóa sau:

2) Đề xuất phương pháp tổng hợp 1-benzyl-2-methyl- và 1-benzyl-3-


methylcyclohexane, đi từ 4-methylcyclohexanone, benzyl alcohol,
toluene và các tác nhân cần thiết khác.
3) Điều chế 3-phenylcyclohexanone và 2-phenylcyclohexadiene-1,3 từ
cyclohexanone, bromobenzene cùng các tác nhân cần thiết khác.
4) Từ acetone và các tác nhân cần thiết, hãy điều chế 3,3-dimethyl-2-
oxobutanal. Điều gì sẽ xảy ra khi xử lí chất này với dung dịch potassium
hydroxide đặc nóng?
5) Đề xuất sơ đồ tổng hợp dicarboxylic acid không no A từ cyclohexanol,
triphenylphosphine, và ethyl ester của bromoacetic acid, cùng các tác
nhân cần thiết khác.

6) Điều chế 1,3,3-trideutero-1-methylcyclopentane từ cyclopentanone,


methyl iodide, cùng các tác nhân cần thiết khác.

254
Hướng dẫn
1)

2)

3)

255
4)

5)

6)

256
10. Cơ chế phản ứng
Đề xuất cơ chế cho các phản ứng sau:
a)

b)

c)

d)

257
Hướng dẫn
a)

b)

c)

d)

258
11. Cơ chế phản ứng
Đề xuất cơ chế cho các phản ứng sau:
a)

b)

c)

d)

e)

259
Hướng dẫn
a)

b)

c)

d)

260
e)

261
12. Cơ chế phản ứng
Đề xuất cơ chế cho các phản ứng sau:
a)

b)

c)

d)

e)

262
Hướng dẫn
a)

b)

263
c)

d)

e)

264
13. Cơ chế phản ứng
Đề xuất cơ chế cho các phản ứng sau:
a)

b)

c)

d)

265
Hướng dẫn
a)

b)

c)

266
d)

267
14. Cơ chế phản ứng
Tiến hành phản ứng giữa 2,2-dibromohexanecarboxylic acid với 4 mol BuLi
trong THF ở -78oC thu được chất A rất hoạt động. Sau đó người ta thêm từng
giọt benzene đã được acid hóa (với sự có mặt của chất hút ẩm là silicagel)
vào môi trường phàn ứng, tiếp theo cho từng giọt chất lỏng B vào và đun
nóng thì người ta phân lập được chất cuối có công thức như sau. Đề nghị cơ
chế của các phản ứng đã xảy ra.

Hướng dẫn

268

You might also like